Download as pdf or txt
Download as pdf or txt
You are on page 1of 102

火箭学院 火速提分

新页码 1
火箭学院 火速提分
新页码 2
火箭学院 火速提分
新页码 3
火箭学院 火速提分
新页码 4
火箭学院 火速提分
新页码 5
火箭学院 火速提分
新页码 6
火箭学院 火速提分
新页码 7
火箭学院 火速提分
新页码 8
火箭学院 火速提分
新页码 9
火箭学院 火速提分
新页码 10
火箭学院 火速提分
新页码 11
火箭学院 火速提分
新页码 12
火箭学院 火速提分
新页码 13
火箭学院 火速提分
新页码 14
火箭学院 火速提分
新页码 15
火箭学院 火速提分
新页码 16
火箭学院 火速提分
新页码 17
火箭学院 火速提分
新页码 18
火箭学院 火速提分
新页码 19
火箭学院 火速提分
新页码 20
火箭学院 火速提分
新页码 21
火箭学院 火速提分
新页码 22
火箭学院 火速提分
新页码 23

AP® Statistics
2002 Free-Response Questions

The materials included in these files are intended for use by AP teachers for course
and exam preparation in the classroom; permission for any other use must be
sought from the Advanced Placement Program®. Teachers may reproduce them, in
whole or in part, in limited quantities, for face-to-face teaching purposes but may
not mass distribute the materials, electronically or otherwise. These materials and
any copies made of them may not be resold, and the copyright notices must be
retained as they appear here. This permission does not apply to any third-party
copyrights contained herein.

These materials were produced by Educational Testing Service® (ETS®), which develops and administers the examinations of the Advanced Placement
Program for the College Board. The College Board and Educational Testing Service (ETS) are dedicated to the principle of equal opportunity, and their
programs, services, and employment policies are guided by that principle.

The College Board is a national nonprofit membership association dedicated to preparing, inspiring, and connecting students to college and opportunity.
Founded in 1900, the association is composed of more than 4,200 schools, colleges, universities, and other educational organizations. Each year, the
College Board serves over three million students and their parents, 22,000 high schools, and 3,500 colleges, through major programs and services in
college admission, guidance, assessment, financial aid, enrollment, and teaching and learning. Among its best-known programs are the SAT®, the
PSAT/NMSQT®, and the Advanced Placement Program® (AP®). The College Board is committed to the principles of equity and
excellence, and that commitment is embodied in all of its programs, services, activities, and concerns.

Copyright © 2002 by College Entrance Examination Board. All rights reserved. College Board, Advanced Placement Program, AP, SAT, and the acorn logo
are registered trademarks of the College Entrance Examination Board. APIEL is a trademark owned by the College Entrance Examination Board. PSAT/NMSQT is a
registered trademark jointly owned by the College Entrance Examination Board and the National Merit Scholarship Corporation.
Educational Testing Service and ETS are registered trademarks of Educational Testing Service.
火箭学院 火速提分
新页码 24

2002 AP® STATISTICS FREE-RESPONSE QUESTIONS

Formulas begin on page 3.


Questions begin on page 6.
Tables begin on page 11.

-2-
火箭学院 火速提分
新页码 25

2002 AP® STATISTICS FREE-RESPONSE QUESTIONS

Formulas

(I) Descriptive Statistics

∑ xi
x =
n

sx =
1
n −1
∑ xi − x
2
3 8

3n1 − 18s21 + 3n2 − 18s22


sp =
3n1 − 18 + 3n2 − 18
y$ = b0 + b1 x

b1 =
3
∑ xi − x y i − y 83 8
3
∑ xi − x 82
b0 = y − b1 x

1 x − x   yi − y 
r =
n −1
∑ i
sx    sy 
sy
b1 = r
sx

3
∑ yi − y$ i 82
sb = n−2
1
3
∑ xi − x 82

-3-
火箭学院 火速提分
新页码 26

2002 AP® STATISTICS FREE-RESPONSE QUESTIONS

(II) Probability

P( A ∪ B) = P( A) + P( B) − P( A ∩ B)

P( A ∩ B )
P( A B ) =
P( B )

E ( X ) = µ x = ∑ xi pi

3
Var( X ) = s 2x = ∑ xi − µ x 82 pi

If X has a binomial distribution with parameters n and p, then:

 n p k (1 − p)n − k
P( X = k ) =
 k
µ x = np

s x = np(1 − p)

µ$ = p
p

p(1 − p)
s p$ =
n

If x is the mean of a random sample of size n from an infinite


population with mean µ and standard deviation s , then:

µx = µ

sx = s
n

-4-
火箭学院 火速提分
新页码 27

2002 AP® STATISTICS FREE-RESPONSE QUESTIONS

(III) Inferential Statistics

statistic - parameter
Standardized test statistic:
standard deviation of statistic

Confidence interval: statistic – (critical value ) œ (standard deviation of statistic)

Single-Sample

Standard Deviation
Statistic
of Statistic
σ
Sample Mean n

p(1 − p)
Sample Proportion n

Two-Sample

Standard Deviation
Statistic
of Statistic

Difference of σ 12 σ 22
sample means +
n1 n2

Special case when σ 1 = σ 2

1 1
σ +
n1 n2

Difference of p1 (1 − p1 ) p (1 − p2 )
sample proportions + 2
n1 n2

Special case when p1 = p2

1 6
p 1− p
1
+
1
n1 n2

Chi-square test statistic = ∑


0observed − expected5 2

expected

-5-
火箭学院 火速提分
新页码 28

2002 AP® STATISTICS FREE-RESPONSE QUESTIONS

STATISTICS
SECTION II
Part A
Questions 1-5
Spend about 65 minutes on this part of the exam.
Percent of Section II grade—75

Directions: Show all your work. Indicate clearly the methods you use, because you will be graded on the
correctness of your methods as well as on the accuracy of your results and explanation.

1. In 1915 Einstein’s theory predicted that the curvature of space, denoted by γ , was 1, while Newtonian theory
predicted it was 0. Since 1915 scientists have repeatedly found estimates of γ using various methods and
procedures. Each estimate has a margin of error. The figure below displays

(estimate ± margin of error)

from each of 21 experiments.

(a) Based on the display on page 6, describe how the precision of the estimates of γ has changed over time.
(b) Write a few sentences describing the strength of evidence the experiments provide for the claim from
Newtonian theory that g = 0. Your response must include justification based on the display.
(c) Write a few sentences describing the strength of evidence the experiments provide for the claim from
Einstein’s theory that g = 1. Your response must include justification based on the display.

Copyright © 2002 by College Entrance Examination Board. All rights reserved.


Advanced Placement Program and AP are registered trademarks of the College Entrance Examination Board.

6 GO ON TO THE NEXT PAGE.


火箭学院 火速提分
新页码 29

2002 AP® STATISTICS FREE-RESPONSE QUESTIONS

2. A manufacturer of boots plans to conduct an experiment to compare a new method of waterproofing to


the current method. The appearance of the boots is not changed by either method. The company recruits
100 volunteers in Seattle, where it rains frequently, to wear the boots as they normally would for 6 months.
At the end of the 6 months, the boots will be returned to the company to be evaluated for water damage.
(a) Describe a design for this experiment that uses the 100 volunteers. Include a few sentences on how it
would be implemented.
(b) Could your design be double blind? Explain.

3. There are 4 runners on the New High School team. The team is planning to participate in a race in which
each runner runs a mile. The team time is the sum of the individual times for the 4 runners. Assume that the
individual times of the 4 runners are all independent of each other. The individual times, in minutes, of the
runners in similar races are approximately normally distributed with the following means and standard
deviations.

Mean Standard Deviation


Runner 1 4.9 0.15
Runner 2 4.7 0.16
Runner 3 4.5 0.14
Runner 4 4.8 0.15

(a) Runner 3 thinks that he can run a mile in less than 4.2 minutes in the next race. Is this likely to happen?
Explain.
(b) The distribution of possible team times is approximately normal. What are the mean and standard deviation
of this distribution?
(c) Suppose the team’s best time to date is 18.4 minutes. What is the probability that the team will beat its own
best time in the next race?

Copyright © 2002 by College Entrance Examination Board. All rights reserved.


Advanced Placement Program and AP are registered trademarks of the College Entrance Examination Board.

7 GO ON TO THE NEXT PAGE.


火箭学院 火速提分
新页码 30

2002 AP® STATISTICS FREE-RESPONSE QUESTIONS

4. Commercial airlines need to know the operating cost per hour of flight for each plane in their fleet. In a study
of the relationship between operating cost per hour and number of passenger seats, investigators computed the
regression of operating cost per hour on the number of passenger seats. The 12 sample aircraft used in the study
included planes with as few as 216 passenger seats and planes with as many as 410 passenger seats. Operating
cost per hour ranged between $3,600 and $7,800. Some computer output from a regression analysis of these
data is shown below.

Predictor Coef StDev T P


Constant 1136 1226 0.93 0.376
Seats 14.673 4.027 3.64 0.005

S = 845.3 R-Sq = 57.0% R-Sq (adj) = 52.7%

(a) What is the equation of the least squares regression line that describes the relationship between operating
cost per hour and number of passenger seats in the plane? Define any variables used in this equation.
(b) What is the value of the correlation coefficient for operating cost per hour and number of passenger seats in
the plane? Interpret this correlation.
(c) Suppose that you want to describe the relationship between operating cost per hour and number of passenger
seats in the plane for planes only in the range of 250 to 350 seats. Does the line shown in the scatterplot still
provide the best description of the relationship for data in this range? Why or why not?

Copyright © 2002 by College Entrance Examination Board. All rights reserved.


Advanced Placement Program and AP are registered trademarks of the College Entrance Examination Board.

8 GO ON TO THE NEXT PAGE.


火箭学院 火速提分
新页码 31

2002 AP® STATISTICS FREE-RESPONSE QUESTIONS

5. Sleep researchers know that some people are early birds (E), preferring to go to bed by 10 P.M. and arise by
7 A.M., while others are night owls (N), preferring to go to bed after 11 P.M. and arise after 8 A.M. A study was
done to compare dream recall for early birds and night owls. One hundred people of each of the two types were
selected at random and asked to record their dreams for one week. Some of the results are presented below.

Number of Dreams Recalled During the Week Proportion Who Recalled


Group Mean Median Standard No 5 or more
Deviation dreams dreams
Early birds 7.26 6.0 6.94 0.24 0.55
Night owls 9.55 9.5 5.88 0.11 0.69

(a) The researchers believe that night owls may have better dream recall than do early birds.
One parameter of interest to the researchers is the mean number of dreams recalled per week with mE
representing this mean for early birds and mN representing this mean for night owls. The appropriate
hypotheses would then be H 0 : mE - mN = 0 and H a : mE - mN < 0. State two other pairs of hypotheses
that might be used to test the researchers’ belief. Be sure to define the parameter of interest in each case.
(b) Use the data provided to carry out a test of the hypotheses about the mean number of dreams recalled
per week given in the statement of part (a). Do the data support the researchers’ belief ?

Copyright © 2002 by College Entrance Examination Board. All rights reserved.


Advanced Placement Program and AP are registered trademarks of the College Entrance Examination Board.

9 GO ON TO THE NEXT PAGE.


火箭学院 火速提分
新页码 32

2002 AP® STATISTICS FREE-RESPONSE QUESTIONS

STATISTICS
SECTION II
Part B
Question 6
Spend about 25 minutes on this part of the exam.
Percent of Section II grade—25

Directions: Show all your work. Indicate clearly the methods you use, because you will be graded on the
correctness of your methods as well as on the accuracy of your results and explanation.

6. A survey given to a random sample of students at a university included a question about which of two well-
known comedy shows, S or F, students preferred. The students were asked the question, “Do you prefer S or F ?”
The responses are shown below.

Preference
S F Total
185 139 324

(a) Based on the results of this survey, construct and interpret a 95% confidence interval for the proportion of
students in the population who would respond S to the question, “Do you prefer S or F ?”
(b) What is the meaning of “95% confidence” in part (a) ?
(c) In a follow-up survey, a separate group of randomly selected students was asked “Do you prefer F or S ?”
The responses are shown below.

Preference
S F Total
68 88 156

Based on these two surveys, is there evidence that the stated preference depends on the order in which the
comedy shows were listed in the survey question? Justify your answer.
(d) Suppose the test in part (c) indicates that the order in which the shows were listed does make a difference.
185 + 68
Is the pooled value = 0.527 a reasonable estimate for the proportion of students at the university
324 + 156
who would respond S ? If so, justify your answer. If not, what would be a more reasonable estimate?
Explain why.

END OF EXAMINATION

Copyright © 2002 by College Entrance Examination Board. All rights reserved.


Advanced Placement Program and AP are registered trademarks of the College Entrance Examination Board.

10 GO ON TO THE NEXT PAGE.


火箭学院 火速提分
新页码 33

2002 AP® STATISTICS FREE-RESPONSE QUESTIONS

Probability

Table entry for z is the


probability lying below z.
z

Table A Standard normal probabilities

z .00 .01 .02 .03 .04 .05 .06 .07 .08 .09
– 3.4 .0003 .0003 .0003 .0003 .0003 .0003 .0003 .0003 .0003 .0002
– 3.3 .0005 .0005 .0005 .0004 .0004 .0004 .0004 .0004 .0004 .0003
– 3.2 .0007 .0007 .0006 .0006 .0006 .0006 .0006 .0005 .0005 .0005
– 3.1 .0010 .0009 .0009 .0009 .0008 .0008 .0008 .0008 .0007 .0007
– 3.0 .0013 .0013 .0013 .0012 .0012 .0011 .0011 .0011 .0010 .0010
– 2.9 .0019 .0018 .0018 .0017 .0016 .0016 .0015 .0015 .0014 .0014
– 2.8 .0026 .0025 .0024 .0023 .0023 .0022 .0021 .0021 .0020 .0019
– 2.7 .0035 .0034 .0033 .0032 .0031 .0030 .0029 .0028 .0027 .0026
– 2.6 .0047 .0045 .0044 .0043 .0041 .0040 .0039 .0038 .0037 .0036
– 2.5 .0062 .0060 .0059 .0057 .0055 .0054 .0052 .0051 .0049 .0048
– 2.4 .0082 .0080 .0078 .0075 .0073 .0071 .0069 .0068 .0066 .0064
– 2.3 .0107 .0104 .0102 .0099 .0096 .0094 .0091 .0089 .0087 .0084
– 2.2 .0139 .0136 .0132 .0129 .0125 .0122 .0119 .0116 .0113 .0110
– 2.1 .0179 .0174 .0170 .0166 .0162 .0158 .0154 .0150 .0146 .0143
– 2.0 .0228 .0222 .0217 .0212 .0207 .0202 .0197 .0192 .0188 .0183
– 1.9 .0287 .0281 .0274 .0268 .0262 .0256 .0250 .0244 .0239 .0233
– 1.8 .0359 .0351 .0344 .0336 .0329 .0322 .0314 .0307 .0301 .0294
– 1.7 .0446 .0436 .0427 .0418 .0409 .0401 .0392 .0384 .0375 .0367
– 1.6 .0548 .0537 .0526 .0516 .0505 .0495 .0485 .0475 .0465 .0455
– 1.5 .0668 .0655 .0643 .0630 .0618 .0606 .0594 .0582 .0571 .0559
– 1.4 .0808 .0793 .0778 .0764 .0749 .0735 .0721 .0708 .0694 .0681
– 1.3 .0968 .0951 .0934 .0918 .0901 .0885 .0869 .0853 .0838 .0823
– 1.2 .1151 .1131 .1112 .1093 .1075 .1056 .1038 .1020 .1003 .0985
– 1.1 .1357 .1335 .1314 .1292 .1271 .1251 .1230 .1210 .1190 .1170
– 1.0 .1587 .1562 .1539 .1515 .1492 .1469 .1446 .1423 .1401 .1379
– 0.9 .1841 .1814 .1788 .1762 .1736 .1711 .1685 .1660 .1635 .1611
– 0.8 .2119 .2090 .2061 .2033 .2005 .1977 .1949 .1922 .1894 .1867
– 0.7 .2420 .2389 .2358 .2327 .2296 .2266 .2236 .2206 .2177 .2148
– 0.6 .2743 .2709 .2676 .2643 .2611 .2578 .2546 .2514 .2483 .2451
– 0.5 .3085 .3050 .3015 .2981 .2946 .2912 .2877 .2843 .2810 .2776
– 0.4 .3446 .3409 .3372 .3336 .3300 .3264 .3228 .3192 .3156 .3121
– 0.3 .3821 .3783 .3745 .3707 .3669 .3632 .3594 .3557 .3520 .3483
– 0.2 .4207 .4168 .4129 .4090 .4052 .4013 .3974 .3936 .3897 .3859
– 0.1 .4602 .4562 .4522 .4483 .4443 .4404 .4364 .4325 .4286 .4247
– 0.0 .5000 .4960 .4920 .4880 .4840 .4801 .4761 .4721 .4681 .4641

11
火箭学院 火速提分
新页码 34

2002 AP® STATISTICS FREE-RESPONSE QUESTIONS

Probability

Table entry for z is the


probability lying below z.
z
Table A (Continued)

z .00 .01 .02 .03 .04 .05 .06 .07 .08 .09
0.0 .5000 .5040 .5080 .5120 .5160 .5199 .5239 .5279 .5319 .5359
0.1 .5398 .5438 .5478 .5517 .5557 .5596 .5636 .5675 .5714 .5753
0.2 .5793 .5832 .5871 .5910 .5948 .5987 .6026 .6064 .6103 .6141
0.3 .6179 .6217 .6255 .6293 .6331 .6368 .6406 .6443 .6480 .6517
0.4 .6554 .6591 .6628 .6664 .6700 .6736 .6772 .6808 .6844 .6879
0.5 .6915 .6950 .6985 .7019 .7054 .7088 .7123 .7157 .7190 .7224
0.6 .7257 .7291 .7324 .7357 .7389 .7422 .7454 .7486 .7517 .7549
0.7 .7580 .7611 .7642 .7673 .7704 .7734 .7764 .7794 .7823 .7852
0.8 .7881 .7910 .7939 .7967 .7995 .8023 .8051 .8078 .8106 .8133
0.9 .8159 .8186 .8212 .8238 .8264 .8289 .8315 .8340 .8365 .8389
1.0 .8413 .8438 .8461 .8485 .8508 .8531 .8554 .8577 .8599 .8621
1.1 .8643 .8665 .8686 .8708 .8729 .8749 .8770 .8790 .8810 .8830
1.2 .8849 .8869 .8888 .8907 .8925 .8944 .8962 .8980 .8997 .9015
1.3 .9032 .9049 .9066 .9082 .9099 .9115 .9131 .9147 .9162 .9177
1.4 .9192 .9207 .9222 .9236 .9251 .9265 .9279 .9292 .9306 .9319
1.5 .9332 .9345 .9357 .9370 .9382 .9394 .9406 .9418 .9429 .9441
1.6 .9452 .9463 .9474 .9484 .9495 .9505 .9515 .9525 .9535 .9545
1.7 .9554 .9564 .9573 .9582 .9591 .9599 .9608 .9616 .9625 .9633
1.8 .9641 .9649 .9656 .9664 .9671 .9678 .9686 .9693 .9699 .9706
1.9 .9713 .9719 .9726 .9732 .9738 .9744 .9750 .9756 .9761 .9767
2.0 .9772 .9778 .9783 .9788 .9793 .9798 .9803 .9808 .9812 .9817
2.1 .9821 .9826 .9830 .9834 .9838 .9842 .9846 .9850 .9854 .9857
2.2 .9861 .9864 .9868 .9871 .9875 .9878 .9881 .9884 .9887 .9890
2.3 .9893 .9896 .9898 .9901 .9904 .9906 .9909 .9911 .9913 .9916
2.4 .9918 .9920 .9922 .9925 .9927 .9929 .9931 .9932 .9934 .9936
2.5 .9938 .9940 .9941 .9943 .9945 .9946 .9948 .9949 .9951 .9952
2.6 .9953 .9955 .9956 .9957 .9959 .9960 .9961 .9962 .9963 .9964
2.7 .9965 .9966 .9967 .9968 .9969 .9970 .9971 .9972 .9973 .9974
2.8 .9974 .9975 .9976 .9977 .9977 .9978 .9979 .9979 .9980 .9981
2.9 .9981 .9982 .9982 .9983 .9984 .9984 .9985 .9985 .9986 .9986
3.0 .9987 .9987 .9987 .9988 .9988 .9989 .9989 .9989 .9990 .9990
3.1 .9990 .9991 .9991 .9991 .9992 .9992 .9992 .9992 .9993 .9993
3.2 .9993 .9993 .9994 .9994 .9994 .9994 .9994 .9995 .9995 .9995
3.3 .9995 .9995 .9995 .9996 .9996 .9996 .9996 .9996 .9996 .9997
3.4 .9997 .9997 .9997 .9997 .9997 .9997 .9997 .9997 .9997 .9998

12
火箭学院 火速提分
新页码 35

2002 AP® STATISTICS FREE-RESPONSE QUESTIONS

Table entry for p and


C is the point t* with Probability p
probability p lying
above it and
probability C lying
between −t * and t*.

t*

Table B t distribution critical values

Tail probability p
df .25 .20 .15 .10 .05 .025 .02 .01 .005 .0025 .001 .0005
1 1.000 1.376 1.963 3.078 6.314 12.71 15.89 31.82 63.66 127.3 318.3 636.6
2 .816 1.061 1.386 1.886 2.920 4.303 4.849 6.965 9.925 14.09 22.33 31.60
3 .765 .978 1.250 1.638 2.353 3.182 3.482 4.541 5.841 7.453 10.21 12.92
4 .741 .941 1.190 1.533 2.132 2.776 2.999 3.747 4.604 5.598 7.173 8.610
5 .727 .920 1.156 1.476 2.015 2.571 2.757 3.365 4.032 4.773 5.893 6.869
6 .718 .906 1.134 1.440 1.943 2.447 2.612 3.143 3.707 4.317 5.208 5.959
7 .711 .896 1.119 1.415 1.895 2.365 2.517 2.998 3.499 4.029 4.785 5.408
8 .706 .889 1.108 1.397 1.860 2.306 2.449 2.896 3.355 3.833 4.501 5.041
9 .703 .883 1.100 1.383 1.833 2.262 2.398 2.821 3.250 3.690 4.297 4.781
10 .700 .879 1.093 1.372 1.812 2.228 2.359 2.764 3.169 3.581 4.144 4.587
11 .697 .876 1.088 1.363 1.796 2.201 2.328 2.718 3.106 3.497 4.025 4.437
12 .695 .873 1.083 1.356 1.782 2.179 2.303 2.681 3.055 3.428 3.930 4.318
13 .694 .870 1.079 1.350 1.771 2.160 2.282 2.650 3.012 3.372 3.852 4.221
14 .692 .868 1.076 1.345 1.761 2.145 2.264 2.624 2.977 3.326 3.787 4.140
15 .691 .866 1.074 1.341 1.753 2.131 2.249 2.602 2.947 3.286 3.733 4.073
16 .690 .865 1.071 1.337 1.746 2.120 2.235 2.583 2.921 3.252 3.686 4.015
17 .689 .863 1.069 1.333 1.740 2.110 2.224 2.567 2.898 3.222 3.646 3.965
18 .688 .862 1.067 1.330 1.734 2.101 2.214 2.552 2.878 3.197 3.611 3.922
19 .688 .861 1.066 1.328 1.729 2.093 2.205 2.539 2.861 3.174 3.579 3.883
20 .687 .860 1.064 1.325 1.725 2.086 2.197 2.528 2.845 3.153 3.552 3.850
21 .686 .859 1.063 1.323 1.721 2.080 2.189 2.518 2.831 3.135 3.527 3.819
22 .686 .858 1.061 1.321 1.717 2.074 2.183 2.508 2.819 3.119 3.505 3.792
23 .685 .858 1.060 1.319 1.714 2.069 2.177 2.500 2.807 3.104 3.485 3.768
24 .685 .857 1.059 1.318 1.711 2.064 2.172 2.492 2.797 3.091 3.467 3.745
25 .684 .856 1.058 1.316 1.708 2.060 2.167 2.485 2.787 3.078 3.450 3.725
26 .684 .856 1.058 1.315 1.706 2.056 2.162 2.479 2.779 3.067 3.435 3.707
27 .684 .855 1.057 1.314 1.703 2.052 2.158 2.473 2.771 3.057 3.421 3.690
28 .683 .855 1.056 1.313 1.701 2.048 2.154 2.467 2.763 3.047 3.408 3.674
29 .683 .854 1.055 1.311 1.699 2.045 2.150 2.462 2.756 3.038 3.396 3.659
30 .683 .854 1.055 1.310 1.697 2.042 2.147 2.457 2.750 3.030 3.385 3.646
40 .681 .851 1.050 1.303 1.684 2.021 2.123 2.423 2.704 2.971 3.307 3.551
50 .679 .849 1.047 1.299 1.676 2.009 2.109 2.403 2.678 2.937 3.261 3.496
60 .679 .848 1.045 1.296 1.671 2.000 2.099 2.390 2.660 2.915 3.232 3.460
80 .678 .846 1.043 1.292 1.664 1.990 2.088 2.374 2.639 2.887 3.195 3.416
100 .677 .845 1.042 1.290 1.660 1.984 2.081 2.364 2.626 2.871 3.174 3.390
1000 .675 .842 1.037 1.282 1.646 1.962 2.056 2.330 2.581 2.813 3.098 3.300
⬁ .674 .841 1.036 1.282 1.645 1.960 2.054 2.326 2.576 2.807 3.091 3.291

50% 60% 70% 80% 90% 95% 96% 98% 99% 99.5% 99.8% 99.9%
Confidence level C

13
火箭学院 火速提分
新页码 36

2002 AP® STATISTICS FREE-RESPONSE QUESTIONS

Probability p
Table entry for p is the point
( χ 2 ) with probability p lying
above it.

(χ2 )

Table C χ 2 critical values


Tail probability p
df .25 .20 .15 .10 .05 .025 .02 .01 .005 .0025 .001 .0005
1 1.32 1.64 2.07 2.71 3.84 5.02 5.41 6.63 7.88 9.14 10.83 12.12
2 2.77 3.22 3.79 4.61 5.99 7.38 7.82 9.21 10.60 11.98 13.82 15.20
3 4.11 4.64 5.32 6.25 7.81 9.35 9.84 11.34 12.84 14.32 16.27 17.73
4 5.39 5.99 6.74 7.78 9.49 11.14 11.67 13.28 14.86 16.42 18.47 20.00
5 6.63 7.29 8.12 9.24 11.07 12.83 13.39 15.09 16.75 18.39 20.51 22.11
6 7.84 8.56 9.45 10.64 12.59 14.45 15.03 16.81 18.55 20.25 22.46 24.10
7 9.04 9.80 10.75 12.02 14.07 16.01 16.62 18.48 20.28 22.04 24.32 26.02
8 10.22 11.03 12.03 13.36 15.51 17.53 18.17 20.09 21.95 23.77 26.12 27.87
9 11.39 12.24 13.29 14.68 16.92 19.02 19.68 21.67 23.59 25.46 27.88 29.67
10 12.55 13.44 14.53 15.99 18.31 20.48 21.16 23.21 25.19 27.11 29.59 31.42
11 13.70 14.63 15.77 17.28 19.68 21.92 22.62 24.72 26.76 28.73 31.26 33.14
12 14.85 15.81 16.99 18.55 21.03 23.34 24.05 26.22 28.30 30.32 32.91 34.82
13 15.98 16.98 18.20 19.81 22.36 24.74 25.47 27.69 29.82 31.88 34.53 36.48
14 17.12 18.15 19.41 21.06 23.68 26.12 26.87 29.14 31.32 33.43 36.12 38.11
15 18.25 19.31 20.60 22.31 25.00 27.49 28.26 30.58 32.80 34.95 37.70 39.72
16 19.37 20.47 21.79 23.54 26.30 28.85 29.63 32.00 34.27 36.46 39.25 41.31
17 20.49 21.61 22.98 24.77 27.59 30.19 31.00 33.41 35.72 37.95 40.79 42.88
18 21.60 22.76 24.16 25.99 28.87 31.53 32.35 34.81 37.16 39.42 42.31 44.43
19 22.72 23.90 25.33 27.20 30.14 32.85 33.69 36.19 38.58 40.88 43.82 45.97
20 23.83 25.04 26.50 28.41 31.41 34.17 35.02 37.57 40.00 42.34 45.31 47.50
21 24.93 26.17 27.66 29.62 32.67 35.48 36.34 38.93 41.40 43.78 46.80 49.01
22 26.04 27.30 28.82 30.81 33.92 36.78 37.66 40.29 42.80 45.20 48.27 50.51
23 27.14 28.43 29.98 32.01 35.17 38.08 38.97 41.64 44.18 46.62 49.73 52.00
24 28.24 29.55 31.13 33.20 36.42 39.36 40.27 42.98 45.56 48.03 51.18 53.48
25 29.34 30.68 32.28 34.38 37.65 40.65 41.57 44.31 46.93 49.44 52.62 54.95
26 30.43 31.79 33.43 35.56 38.89 41.92 42.86 45.64 48.29 50.83 54.05 56.41
27 31.53 32.91 34.57 36.74 40.11 43.19 44.14 46.96 49.64 52.22 55.48 57.86
28 32.62 34.03 35.71 37.92 41.34 44.46 45.42 48.28 50.99 53.59 56.89 59.30
29 33.71 35.14 36.85 39.09 42.56 45.72 46.69 49.59 52.34 54.97 58.30 60.73
30 34.80 36.25 37.99 40.26 43.77 46.98 47.96 50.89 53.67 56.33 59.70 62.16
40 45.62 47.27 49.24 51.81 55.76 59.34 60.44 63.69 66.77 69.70 73.40 76.09
50 56.33 58.16 60.35 63.17 67.50 71.42 72.61 76.15 79.49 82.66 86.66 89.56
60 66.98 68.97 71.34 74.40 79.08 83.30 84.58 88.38 91.95 95.34 99.61 102.7
80 88.13 90.41 93.11 96.58 101.9 106.6 108.1 112.3 116.3 120.1 124.8 128.3
100 109.1 111.7 114.7 118.5 124.3 129.6 131.1 135.8 140.2 144.3 149.4 153.2

14
火箭学院 火速提分
新页码 37
火箭学院 火速提分
新页码 38

AP® Statistics
2002 Scoring Guidelines

The materials included in these files are intended for use by AP teachers for course
and exam preparation in the classroom; permission for any other use must be
sought from the Advanced Placement Program®. Teachers may reproduce them, in
whole or in part, in limited quantities, for face-to-face teaching purposes but may
not mass distribute the materials, electronically or otherwise. These materials and
any copies made of them may not be resold, and the copyright notices must be
retained as they appear here. This permission does not apply to any third-party
copyrights contained herein.

These materials were produced by Educational Testing Service® (ETS®), which develops and administers the examinations of the Advanced Placement
Program for the College Board. The College Board and Educational Testing Service (ETS) are dedicated to the principle of equal opportunity, and their
programs, services, and employment policies are guided by that principle.

The College Board is a national nonprofit membership association dedicated to preparing, inspiring, and connecting students to college and opportunity.
Founded in 1900, the association is composed of more than 4,200 schools, colleges, universities, and other educational organizations. Each year, the
College Board serves over three million students and their parents, 22,000 high schools, and 3,500 colleges, through major programs and services in
college admission, guidance, assessment, financial aid, enrollment, and teaching and learning. Among its best-known programs are the SAT®, the
PSAT/NMSQT®, and the Advanced Placement Program® (AP®). The College Board is committed to the principles of equity and
excellence, and that commitment is embodied in all of its programs, services, activities, and concerns.

Copyright © 2002 by College Entrance Examination Board. All rights reserved. College Board, Advanced Placement Program, AP, SAT, and the acorn logo
are registered trademarks of the College Entrance Examination Board. APIEL is a trademark owned by the College Entrance Examination Board. PSAT/NMSQT is a
registered trademark jointly owned by the College Entrance Examination Board and the National Merit Scholarship Corporation.
Educational Testing Service and ETS are registered trademarks of Educational Testing Service.
火箭学院 火速提分
新页码 39

AP® STATISTICS
2002 SCORING GUIDELINES

Question 1

Solution

Part (a):

The precision of the estimates of γ has gotten better over time. This is indicated by the
fact that the intervals
value ± (margin of error)
shown in the figure become narrower over time, indicating that the margin of error is
getting smaller.

Part (b):

The value of γ = 0 is not included in any of the 21 intervals. This indicates that 0 is not a
plausible value for γ . There is no support for Newton's theory.

Part (c):

The support for Einstein's theory that γ = 1 is quite strong. Most of the intervals contain
the value 1, and the more recent intervals, where the precision is greater, suggest that the
value of γ is at least very close to 1.

Scoring

Part (a) is scored as incorrect (I), partially correct (P), or essentially correct (E). The
response is essentially correct if the student indicates that precision is increasing. (If the
student also explains how he or she can tell this from the figure, this is a plus.)

If the student incorrectly says that precision is decreasing, but gives a good explanation
that is tied to the figure, the response is scored as partially correct.

Part (b) is scored as incorrect (I), partially correct (P), or essentially correct (E). To be
scored as essentially correct, the response must say that there is no (or very weak) support
and give a valid reason for this conclusion based on the intervals in the figure.

If the student states only that there is no (or weak) support, but does not say how this
follows from the intervals in the figure, the response is scored as partially correct.

Part (c) is scored as incorrect (I), partially correct (P), or essentially correct (E). To be
scored as essentially correct, the response must say that there is strong support and give a
valid reason for this conclusion based on the intervals in the figure.

If the student states only that there is strong support, but does not say how this follows
from the intervals in the figure, the response is scored as partially correct.

Copyright © 2002 by College Entrance Examination Board. All rights reserved.


Advanced Placement Program and AP are registered trademarks of the College Entrance Examination Board.

2
火箭学院 火速提分
新页码 40

AP® STATISTICS
2002 SCORING GUIDELINES

Question 1 (cont’d.)

4 Complete Response

All three parts essentially correct.

3 Substantial Response

Two parts essentially correct and one part partially correct.

2 Developing Response

Two parts essentially correct and no parts partially correct.


OR
One part essentially correct and two parts partially correct.
OR
Three parts partially correct.

1 Minimal Response

One part essentially correct and either zero or one parts partially correct.
OR
No parts essentially correct and two parts partially correct.

Copyright © 2002 by College Entrance Examination Board. All rights reserved.


Advanced Placement Program and AP are registered trademarks of the College Entrance Examination Board.

3
火箭学院 火速提分
新页码 41

AP® STATISTICS
2002 SCORING GUIDELINES
Question 2

Solution

Part (a):
A paired design is used in which each subject receives a pair of boots where one boot is treated with the
new method and the other with the current method.
Subjects should be randomly assigned to one of two groups. Group 1 would have the new
method applied to the right boot; group 2 would have the new method applied to the left boot.
OR
For each subject, whether the new method is applied to the right or left boot is determined at
random.
OR
A crossover design is used in which each subject receives a pair of boots, both of which were treated
with one treatment. The boots are used for three months and then exchanged for a second pair of boots,
both of which were treated with the other treatment. These boots are then used for the next three months.
Subjects should be randomly assigned to one of two groups. One group receives boots with the
new treatment first and the other group receives boots with the current method first.

NOTE: Additional appropriate blocking schemes are considered extraneous.

Part (b):
The design could be double blind, as long as both the subjects and the person evaluating the boots for
water damage do not know which boots were treated with the new method and which were treated with
the current method.

NOTE: If the student does something unexpected in part (a) and gives a design that actually cannot be
double blind, then part (b) could be considered correct provided the response explains why the design
could not be double blind.

Scoring
A student response is scored as E (essentially correct), P (partially correct), or I (incorrect) for each of the
following key elements:
1. Design
• E - paired design (may be described as blocking on individual) or crossover design
• P - 2 or more groups (e.g., Completely Randomized Design)
• I - no grouping or grouping with no treatments specified
2. Implementation: Randomization appropriate to the design
• E - Written description of appropriate randomization
• P - Incomplete or incorrect description of randomization
• I - No description of randomization
NOTE: (1) Diagram alone can be scored at most a P.
(2) The randomization must apply to the allocation or assignment of subjects to the
treatment groups or the allocation of treatments to the subjects.
(3) Randomization to select the 100 volunteers without assignment to the treatment
groups is scored an I.

Copyright © 2002 by College Entrance Examination Board. All rights reserved.


Advanced Placement Program and AP are registered trademarks of the College Entrance Examination Board.

4
火箭学院 火速提分
新页码 42

AP® STATISTICS
2002 SCORING GUIDELINES
Question 2 (cont’d.)

3. Double blind: Explanation in parts (a) and/or (b) that shows understanding of what it means for
an experiment to be double blind.
• E - response indicates that blinding applies to both the evaluator and subjects.
• P - response recognizes that blinding applies to the subjects and at least one other party,
whether or not they think that this can be accomplished; the other party may not be
correctly identified.
• I - response fails to recognize that both the subject and another party must be blinded or is
missing or irrelevant.
Score as Design - Randomization - Double Blind

4 Complete Response

EEE

3 Substantial Response

Any one of the following combinations:

EEP PEE PEP*


EEI
EPE

2 Developing Response

Any one of the following combinations:

EPP PEI IEE PEP *


EPI PPE IPE
EIE PPP
EIP PIE

1 Minimal Response

Any one of the following combinations:

EII PPI IEP


IEI PIP
IIE IPP

Copyright © 2002 by College Entrance Examination Board. All rights reserved.


Advanced Placement Program and AP are registered trademarks of the College Entrance Examination Board.

5
火箭学院 火速提分
新页码 43

AP® STATISTICS
2002 SCORING GUIDELINES
Question 2 (cont’d.)

0 No Credit

PII III
IPI
IIP

* P E P may be scored as either a 2 or a 3:


(1) If the description of the randomization only says, “Randomly allocate”, then score P E P a 2.
(2) If the description of the randomization says, “Randomly allocate”, but also contains greater detail
about the randomization or the inclusion of blocking in the design or other statistical thinking, then
score P E P a 3.

Copyright © 2002 by College Entrance Examination Board. All rights reserved.


Advanced Placement Program and AP are registered trademarks of the College Entrance Examination Board.

6
火箭学院 火速提分
新页码 44

AP® STATISTICS
2002 SCORING GUIDELINES

Question 3

Solution

Part (a):

For runner 3

P (time < 4.2) = P æ z <


è
4.2 - 4.5
.14 )= P ( z < -2.14) = 0.0162 ( from table)
OR

P (time < 4.2) = 0.0160622279 (from Calculator)

It is possible but unlikely that runner 3 will run a mile in less than 4.2 minutes on the next
race. Based on his running time distribution, we would expect that he would have times
less than 4.2 minutes less than 2 times in 100 races in the long run.

OR

It is possible but unlikely that runner 3 will run a mile in less than 4.2 minutes on the next
race because 4.2 is more than 2 standard deviations below the mean. Since the running
time has a normal distribution, it is unlikely to be more than 2 standard deviations below
the mean.

Part (b):
mT = m1 + m2 + m3 + m4 = 4.9 + 4.7 + 4.5 + 4.8 = 18.9

The runners’ times are independently distributed, therefore


sT2 = s12 + s 22 + s 32 + s 42 = (.15)2 + (.16)2 + (.14)2 + (.15)2 = 0.0902

sT = .0902 = 0.3003

Part (c):

P (team time < 18.4) = P æ z <


è
18.4 - 18.9
.3003 )
= P ( z < -1.67) = 0.0475 ( from table)
OR

P(team time < 18.4) = 0.0479561904 (from Calculator)

Copyright © 2002 by College Entrance Examination Board. All rights reserved.


Advanced Placement Program and AP are registered trademarks of the College Entrance Examination Board.

7
火箭学院 火速提分
新页码 45

AP® STATISTICS
2002 SCORING GUIDELINES

Question 3 (cont’d.)

Scoring

Each part is scored as essentially correct (E), partially correct (P), or incorrect (I).

Part (a) is essentially correct if:


the probability is calculated correctly, it is correctly assessed as unlikely, and the
justification is acceptable
OR
the student does not compute the probability, but appeals to the fact that a time of 4.2
minutes is more than 2 standard deviations below the mean of a normal distribution and
then uses this information to reach a conclusion with appropriate communication.

Part (a) is partially correct if:


the probability computed is not correct (for example, P(z > −2.14) or P(z < +2.14) might
be computed), but the given probability is correctly assessed
OR
an argument is based on the number of standard deviations from the mean without
invoking normality.

Part (b) is essentially correct if both the mean and the standard deviation of the team time
distribution are correctly computed (except for purely arithmetic mistakes).

Part (b) is partially correct if only one of these is correctly computed (except for purely
arithmetic mistakes).

CAUTION: A standard deviation of .3 (numerically correct) can arise from this incorrect
(.15+.16+.14+.15)
calculation: = 0.3
4

Part (c) is essentially correct if the probability is correctly calculated using a mean which is
either correct or carried from (b) as well as a standard deviation which is either correct or carried
from (b).

Part (c) is partially correct if:

both the mean and standard deviation are correct or carried from (b), but the computed
probability is incorrect
OR
the mean or standard deviation is incorrectly derived from (b) but the subsequent
probability calculation is correct.

Copyright © 2002 by College Entrance Examination Board. All rights reserved.


Advanced Placement Program and AP are registered trademarks of the College Entrance Examination Board.

8
火箭学院 火速提分
新页码 46

AP® STATISTICS
2002 SCORING GUIDELINES

Question 3 (cont’d.)

4 Complete Response

All three parts essentially correct.

3 Substantial Response

Two parts essentially correct and one part partially correct.

2 Developing Response
Two parts essentially correct and no parts partially correct.
OR
One part essentially correct and two parts partially correct.
OR
One part essentially correct and one part partially correct.
OR
Three parts partially correct.

1 Minimal Response
One part essentially correct and zero parts partially correct.
OR
No parts essentially correct and two parts partially correct.

Copyright © 2002 by College Entrance Examination Board. All rights reserved.


Advanced Placement Program and AP are registered trademarks of the College Entrance Examination Board.

9
火箭学院 火速提分
新页码 47

AP® STATISTICS
2002 SCORING GUIDELINES

Question 4

Solution

Part (a):
Predicted cost = 1136 + 14.673 (number of passenger seats)

OR

yˆ = 1136 + 14.673 x where y = operating cost per hour


and x = number of passenger seats

Part (b):
• The value of the correlation coefficient

r = + 0.570 = 0.755 (r is positive because the scatterplot shows a positive


association)
• The interpretation of correlation

There is a moderate (or strong) positive linear relationship between operating


costs per hour and number of passenger seats.

OR

Fifty-seven percent of the variability in operating cost per hour can be explained
by a linear relationship between cost and number of passenger seats AND the
relationship is positive.

Part (c):
No. The equation of the least-squares regression line is influenced by the three points in
the upper right-hand corner and the two points in the lower left-hand corner of the
scatterplot. The seven remaining points (with number of seats in the 250 to 350 range)
would have a negative correlation. Hence, the slope of the recalculated least-squares
regression line is negative.

Scoring
The student response should include the following elements:
1. the correct equation of the least squares regression line with variables correctly
defined;
2. the correct value for the correlation coefficient;
3. a correct interpretation of the given correlation coefficient; and
4. a complete explanation of why the given least-squares line would not be
appropriate for describing the relationship over the restricted range.

Each element is scored as essentially correct (E), partially correct (P), or incorrect (I).

Copyright © 2002 by College Entrance Examination Board. All rights reserved.


Advanced Placement Program and AP are registered trademarks of the College Entrance Examination Board.

10
火箭学院 火速提分
新页码 48

AP® STATISTICS
2002 SCORING GUIDELINES

Question 4 (cont’d.)

Part (a) addresses the first element.

Element one is:

essentially correct if the solution has the correct equation and variables are
defined correctly.
partially correct if only the equation is correct.
incorrect if the equation is not stated correctly.

Part (b) addresses the second and third elements.

Element two is:

essentially correct if the student’s solution states that r = 0.755.


partially correct if the student’s solution only states that r = ± 0.755.
incorrect if the student states any other value of r including
r = 0.726 (square root of R-Sq (adj)).

Element three is:

essentially correct if the student’s solution

addresses, based on a correct understanding of the correlation coefficient,


three or four of the following:
• type of relationship
• strength
• direction
• context

OR

states, based on a correct understanding of r2:


• that 57 percent of the variability in operating cost per hour can
be explained by a linear relationship between cost and number of
passenger seats

AND

• that the relationship is positive.

Note: If the student gives a correct interpretation of r but then


incorrectly explains r2, this is considered a parallel solution
and is incorrect.

Copyright © 2002 by College Entrance Examination Board. All rights reserved.


Advanced Placement Program and AP are registered trademarks of the College Entrance Examination Board.

11
火箭学院 火速提分
新页码 49

AP® STATISTICS
2002 SCORING GUIDELINES

Question 4 (cont’d.)

partially correct if the student’s solution

addresses exactly two of the following – type of relationship (linear),


strength, direction, and context (based on a correct understanding of the
correlation coefficient).

OR

only states that 57 percent of the variability in operating cost per hour
can be explained by a linear relationship between cost and number of
passenger seats (based on a correct understanding of r2) – BUT – does
not state that the relationship is positive.

NOTE: Element three may be scored essentially or partially correct if the student uses a
reasonable r (between 0 and 1) or R-Sq (adj) value.

Part (c) addresses the fourth element.

Element four is essentially correct if the student’s solution

states that the existing line is not a good fit for the remaining seven points and
correctly explains that the restricted data has a negative correlation or the
recalculated least-squares regression line has a negative slope.

Element four is partially correct if the student’s solution

explains why the existing line is not a good fit for the remaining seven points but
does not communicate that the restricted data has a negative correlation or the
recalculated least-squares regression line has a negative slope.

OR

removes fewer than the specified five points, but gives a correct interpretation of
the effect on the correlation or slope of the least-squares regression line.

Copyright © 2002 by College Entrance Examination Board. All rights reserved.


Advanced Placement Program and AP are registered trademarks of the College Entrance Examination Board.

12
火箭学院 火速提分
新页码 50

AP® STATISTICS
2002 SCORING GUIDELINES

Question 4 (cont’d.)

For elements 1 through 4, essentially correct responses count as one element and partially
correct responses count as one-half of an element.

4 Complete Response

Four elements correct.

3 Substantial Response

Three elements correct.

2 Developing Response

Two elements correct.

1 Minimal Response

One element correct.

If a paper is between two scores (for example, 2 1/2 elements) use a holistic approach to
determine whether to score up or down depending on the strength of the response and quality
of communication.

Copyright © 2002 by College Entrance Examination Board. All rights reserved.


Advanced Placement Program and AP are registered trademarks of the College Entrance Examination Board.

13
火箭学院 火速提分
新页码 51

AP® STATISTICS
2002 SCORING GUIDELINES

Question 5

Solution

Part (a):

Possibilities include:

1. pE = proportion of early birds who recall dreams


pN = proportion of night owls who recall dreams

H0: pE - pN = 0 vs. Ha: pE - pN < 0 OR H0: pE = pN vs. Ha: pE < pN


OR
pE = proportion of early birds who do not recall dreams
pN = proportion of night owls who do not recall dreams

H0: pE - pN = 0 vs. Ha: pE - pN > 0 OR H0: pE = pN vs. Ha: pE > pN

NOTE: Either of these, BUT NOT BOTH, can be used as one of the possibilities
for part (a).

2. pE = proportion of early birds who recall 5 or more dreams


pN = proportion of night owls who recall 5 or more dreams

H0: pE - pN = 0 vs. Ha: pE - pN < 0 OR H0: pE = pN vs. Ha: pE < pN


OR
pE = proportion of early birds who do not recall 5 or more dreams
pN = proportion of night owls who do not recall 5 or more dreams

H0: pE - pN = 0 vs. Ha: pE - pN > 0 OR H0: pE = pN vs. Ha: pE > pN

NOTE: Either of these, BUT NOT BOTH, can be used as one of the possibilities
for part (a).

3. ME = median number of dreams early birds recall


MN = median number of dreams night owls recall

H0: ME - MN = 0 vs. Ha: ME - MN < 0 OR H0: ME = MN vs. Ha: ME < MN

NOTE: 1. A complete response for part (a) requires two pairs of hypotheses.
2. Hypotheses for a chi-square test of homogeneity are not correct
since this is a one-sided test.

Copyright © 2002 by College Entrance Examination Board. All rights reserved.


Advanced Placement Program and AP are registered trademarks of the College Entrance Examination Board.

14
火箭学院 火速提分
新页码 52

AP® STATISTICS
2002 SCORING GUIDELINES

Question 5 (cont’d.)

Part (b):

Part 1: States a correct pair of hypotheses, identifies a correct test (by name or by formula)
and checks appropriate requirements.

mE = mean number of dreams early birds recall


mN = mean number of dreams night owls recall

H 0 : mE = mN H 0 : mE - mN = 0
OR
H a : mE < mN H a : mE - mN < 0

xE - x N - 0
Two-sample t-test t=
sE2 s N2
+
nE n N

Requirements:
1. Problem states that independent random samples were taken.
2. Normal population distributions or large samples. Since these are not normal, we
need to note that nE = 100 and nN = 100 are both large in order to perform the t-test.

OR
xE - x N - 0
Two-sample z-test z=
sE2 s N2
+
nE n N
Requirements:
1. Problem states that independent random samples were taken.
2. Since nE = 100 and nN = 100 are both large, it is OK to perform the
approximate z-test.
OR
xE - x N - 0
Pooled t-test t= where s 2p is the pooled variance.
1 1
sp +
nE n N
Requirements:
1. Problem states that independent random samples were taken.
2. Normal population distributions or large samples. Since these are not normal, we
need to note that nE = 100 and nN = 100 are both large in order to perform the t test.
3. The sample standard deviations sE = 6.94 and sN = 5.88 are reasonably close, so it
is OK to assume that the two population variances are equal, i.e. s E2 = s N2 .

Copyright © 2002 by College Entrance Examination Board. All rights reserved.


Advanced Placement Program and AP are registered trademarks of the College Entrance Examination Board.

15
火箭学院 火速提分
新页码 53

AP® STATISTICS
2002 SCORING GUIDELINES

Question 5 (cont’d.)

Part 2: Correct mechanics, including the value of the test statistic, df, and P-value (or
rejection region)

xE - x N - 0 7.26 - 9.55
t= = = -2.52
sE2 s N2 (6.94) 2 (5.88)2
+ +
nE n N 100 100

So, t = -2.52 df = 192 P-value = 0.006

• It is OK to use conservative df of 99.


• Using t-tables: P-value < 0.01
• Using calculator: t = − 2.517578, P-value = 0.006304, df = 192.799
• Using z-test: P-value = 0.005908
• Using z-table: 0.0059 < P-value < 0.0060
• The pooled t-test results in the same value of t, but a df of 198.

Part 3: Stating a correct conclusion in the context of the problem, using the result of the
statistical test.

Because the P-value is small (or less than an α selected and stated by the student), reject H0.
There is convincing evidence that the mean number of dreams night owls recall is greater for
than the mean number of dreams early birds recall.

Copyright © 2002 by College Entrance Examination Board. All rights reserved.


Advanced Placement Program and AP are registered trademarks of the College Entrance Examination Board.

16
火箭学院 火速提分
新页码 54

AP® STATISTICS
2002 SCORING GUIDELINES

Question 5 (cont’d.)

Scoring

Part (a):
Essentially correct (E) if two distinct pairs of hypotheses are given and the parameters in the
hypotheses are defined.

Partially correct (P) if only one pair of correct hypotheses is given and the parameters in these
hypotheses are defined or if two “approved” pairs of hypotheses are given but the parameters are
poorly defined.

Part (b):
Each of the 3 parts of the hypothesis test is scored either as correct (E) or incorrect (I).
• Because the hypotheses are given in the statement of part (a), they need not be restated here.
However, if wrong hypotheses are stated, then part 1 is scored as incorrect.
• Because the problem states that samples are random, it is OK if a student doesn’t repeat this.
• Some reference to both samples being large is essential.
• For the pooled t- test, some comment on the reasonableness of such an assumption is necessary. It
is not sufficient just to say that population variances are equal.

4 Complete Response
Part (a) essentially correct and all three parts of the hypothesis test correct.

3 Substantial Response
Part (a) essentially correct and two parts of the hypothesis test correct.
OR
Part (a) partially correct or incorrect and three parts of the hypothesis test correct.

Part (a) partially correct and two parts of the hypothesis test correct may be scored either as a
2 or a 3, depending on the overall strength of the paper.

2 Developing Response

Part (a) essentially correct and one part of the hypothesis test correct.
OR
Part (a) incorrect and two parts of the hypothesis test correct.

Part (a) partially correct and two parts of the hypothesis test correct may be scored either as a
2 or a 3, depending on the overall strength of the paper.

1 Minimal Response

Part (a) essentially correct.


OR
Part (a) partially correct and zero or one parts of the hypothesis test correct.
OR
Part (a) is incorrect and one part of the hypothesis test correct.

Copyright © 2002 by College Entrance Examination Board. All rights reserved.


Advanced Placement Program and AP are registered trademarks of the College Entrance Examination Board.

17
火箭学院 火速提分
新页码 55

AP® STATISTICS
2002 SCORING GUIDELINES

Question 6

Solution

Part (a):

p = proportion of students at this university who would respond S to the question, “Do you prefer
S or F?”

Large sample confidence interval for a population proportion.

pˆ (1 - pˆ )
pˆ ± 1.96
n

State and Check Assumptions and Conditions:

Simple random sample (given in the problem stem—need not be mentioned in solution). Also
need large sample with npˆ ³ 5 and n (1 - pˆ ) ³ 5 . Here,

npˆ = 185 n(1 - pˆ ) = 139 are both greater than 5 (or 10)

pˆ (1 - pˆ )
or pˆ ± 3 is entirely in the interval (0,1).
n

We assume the university has at least 10(324) = 3240 students ( N ³ 10n ).

Calculations:

pˆ (1 - pˆ ) (.571) (.429)
pˆ ± 1.96 = 0.571 ± 1.96 = 0.571 ± 0.054 = (0.517, 0.625)
n 324

Calculator solution: (0.5171, 0.62488). The procedure is specified in the stem, but students still need
to check assumptions and conditions.

Interpretation:

Based on this sample, we can be 95 percent confident that the proportion of students at this university
who would respond S to the question, “Do you prefer S or F?” is between 0.517 and 0.625.
OR
We have 95 percent confidence that the interval (0.5171, 0.62488) captures the proportion of
students who would respond “S” to the question, “Do you prefer S or F?”

Copyright © 2002 by College Entrance Examination Board. All rights reserved.


Advanced Placement Program and AP are registered trademarks of the College Entrance Examination Board.

18
火箭学院 火速提分
新页码 56

AP® STATISTICS
2002 SCORING GUIDELINES

Question 6 (cont’d.)

Part (b):

Meaning of Confidence Level:

In repeated sampling, 95 percent of the intervals produced using this method will contain the
proportion of students at this university who would respond S to the question “Do you prefer S or F?”

Part (c):

Approach 1: Hypothesis Test – Two Proportion Z- test

States a Correct Pair of Hypotheses:

H0: p1 - p2 = 0
Ha: p1 - p2 ¹ 0

where
p1 = proportion of students at this university who would respond S with the original
question wording
p2 = proportion of students at this university who would respond S with the revised
question wording

Note: A one-sided test is incorrect.

Name Test and State and Check Assumptions and Conditions:

Identifies a correct test (by name or by formula), and checks appropriate assumptions.

Two sample z-test for proportions

Note: Problem states that samples are random samples, so this does not need to be
addressed in the assumptions.

Large samples: n1 pˆ1 = 185; n1 (1 - pˆ1 ) = 139; n2 pˆ 2 = 68; n2 (1 - pˆ 2 ) = 88


All are greater than 5 (or 10), so the sample sizes are large enough.

Copyright © 2002 by College Entrance Examination Board. All rights reserved.


Advanced Placement Program and AP are registered trademarks of the College Entrance Examination Board.

19
火箭学院 火速提分
新页码 57

AP® STATISTICS
2002 SCORING GUIDELINES

Question 6 (cont’d.)

Calculations:

Correct mechanics, including P-value or rejection region (except for minor arithmetic
errors)

For two sample proportion z-test:

185 68
pˆ1 = = 0.571 pˆ 2 = = 0.436
324 156

185 + 68 253
pˆ = = = 0.527
324 + 156 480

pˆ1 - pˆ 2 0.571 - 0.436 0.135


z= = = = 2.77
pˆ (1 - pˆ ) pˆ (1 - pˆ ) (0.527) (0.473) (0.527) (0.473) 0.0487
+ +
n1 n2 324 156

P-value = 2(.0028) = .0056 from tables

(Calculator: z = 2.776554085, P-value = .0054939656)

If the proportions are not pooled, then z = 2.795 and p = 0.00518.

Conclusion:

Since the P-value (0.0055) is so small, we reject the null hypothesis that the proportions
of this university’s students who would respond S to the two survey questions are equal.
We believe the order in which the choices are given affects the students’ response.

Stating a correct conclusion in the context of the problem, using the result of the
statistical test (i.e., linking the conclusion to the result of the hypothesis test).

If both an α and a P-value are given, the linkage is implied. If no α is given, the solution
must be explicit about the linkage by giving a correct interpretation of the P-value or
explaining how the conclusion follows from the P-value.

If the P-value in part 3 is incorrect but the conclusion is consistent with the computed P-
value, part 4 should be considered as correct.

Copyright © 2002 by College Entrance Examination Board. All rights reserved.


Advanced Placement Program and AP are registered trademarks of the College Entrance Examination Board.

20
火箭学院 火速提分
新页码 58

AP® STATISTICS
2002 SCORING GUIDELINES

Question 6 (cont’d.)

Approach 2: Hypothesis Test – Chi-square Test for Homogeneity

States a correct pair of hypotheses:


H0: population response proportions are the same for the two question wordings
Ha: population response proportions are not the same for the two question
wordings

NOTE: Although the computations are the same, stating the hypotheses in terms of
independence is not correct.

Name Test and State and Check Assumptions and Conditions:

Identifies a correct test (by name or by formula), and checks appropriate assumptions.

Chi-square test for homogeneity

Observed counts
S F row total
Original wording 185 139 324
Revised wording 68 88 156
Column total 253 227 480

Expected counts
S F
Original wording 170.775 153.225
Revised wording 82.225 73.775

NOTE: Problem states that samples are random samples, so this does not need to be
addressed in the assumptions. All expected counts are greater than five, so the sample
sizes are large enough.

Calculations:

Correct mechanics, including P-value or rejection region (except for minor arithmetic
errors)

For chi-square test:

(O - E )2 (185 - 170.775) 2 (88 - 73.775) 2


c2 = å = + ... + = 7.709253
E 170.775 73.775

df = 1 from tables 0.005 < P-value < 0.01

(Calculator: P-value = 0.0054938481)

Copyright © 2002 by College Entrance Examination Board. All rights reserved.


Advanced Placement Program and AP are registered trademarks of the College Entrance Examination Board.

21
火箭学院 火速提分
新页码 59

AP® STATISTICS
2002 SCORING GUIDELINES

Question 6 (cont’d.)

Conclusion:

Since the P-value is so small, we reject the null hypothesis that the proportions of
students at this university who would respond S are the same for the two survey
questions. We believe the order in which the choices are given affects the students’
response.

Stating a correct conclusion in the context of the problem, using the result of the
statistical test (i.e., linking the conclusion to the result of the hypothesis test).

If both an α and a P-value are given, the linkage is implied. If no α is given, the solution
must be explicit about the linkage by giving a correct interpretation of the P-value or
explaining how the conclusion follows from the P-value.

If the P-value in part 3 is incorrect but the conclusion is consistent with the computed P-
value, part 4 should be considered as correct.

Approach 3: Two sample confidence interval

Name Test and State and Check Assumptions and Conditions:

Two-sample confidence interval.

Problem states that samples are random samples, so this does not need to be
addressed in the assumptions.

Large samples:

n1 pˆ1 = 185; n1 (1 - pˆ1 ) = 139; n2 pˆ 2 = 68; n2 (1 - pˆ 2 ) = 88

All are larger than 5 (or 10), so the sample sizes are large enough.

Calculations:

pˆ1 (1 - pˆ1 ) pˆ (1 - pˆ 2 )
( pˆ1 - pˆ 2 ) ± z* + 2
n1 n2

90 percent CI: (0.05565, 0.21453)


95 percent CI: (0.04044, 0.22974)
99 percent CI: (0.01069, 0.25949)

Copyright © 2002 by College Entrance Examination Board. All rights reserved.


Advanced Placement Program and AP are registered trademarks of the College Entrance Examination Board.

22
火箭学院 火速提分
新页码 60

AP® STATISTICS
2002 SCORING GUIDELINES

Question 6 (cont’d.)

Conclusion:

Since the confidence interval does not include 0, there is evidence that the proportions of
students at this university who respond S is not the same for the two question wordings.

Approach 4: (This approach can score at most partially correct for part (c))

One sample confidence interval for p2 .

All checks and assumptions must be included (same as in section (a)).

95 percent CI: (0.35808, 0.51371)

Since this interval does not overlap with the interval computed in part (a), (0.517,0.625) ,
conclude that the proportions of this university’s students who would respond S is not the
same for the two question wordings.

Part (d):

If the sample sizes had been equal, it would be reasonable to combine the data from the
two samples by pooling, which would be equivalent to averaging the two proportions in
this case. But, since the wording of the question makes a difference, and more people
were asked the original version than were asked the revised version, we cannot just pool.

OR

It is only reasonable to pool estimates if they are estimating the same population
parameter. Here the proportion who would respond S differs with the survey question so
the estimates should not be pooled.

Approach 1:

One reasonable approach would be to scale sample 2 up to a sample size of 324 while
maintaining the same sample proportion. To do this, the number of S's in
sample 2 would be multiplied by a factor of 2.076923 (It is OK if the student uses a
factor of 2 for simplicity). This would result in two samples of sizes 324 with
185 S's in sample 1 and 141 S's in sample 2. This would result in an estimate of those
who prefer S of

185 + 141 326


pˆ = = = 0.503
648 648

Copyright © 2002 by College Entrance Examination Board. All rights reserved.


Advanced Placement Program and AP are registered trademarks of the College Entrance Examination Board.

23
火箭学院 火速提分
新页码 61

AP® STATISTICS
2002 SCORING GUIDELINES

Question 6 (cont’d.)

Note: A comparable approach would be to scale sample 1 down to a sample size of 156
by using a multiplier of 0.481481 to obtain

89 + 68 157
pˆ = = = 0.503 This is very close to 0.5.
312 312

Approach 2:

The approach described above is equivalent to just averaging the two proportions, and so
averaging the two given proportions is also an acceptable approach.

185 68
+ 0.571 + 0.436
pˆ = 324 156 = = 0.503
2 2

Note: A weighted average of the two proportions (with weights proportional to sample
size) is equivalent to the given pooled value. If the student rejects the pooled value and
proposes a weighted average of the two sample proportions as an alternative, part (d) is
incorrect.

Scoring for Question 6

Parts (a) and (b) should be read together.

Part (a) is scored as essentially correct (E) if the assumptions are checked, the interval is
computed correctly (except for minor arithmetic errors), and a correct interpretation of
the interval is given.

It is partially correct (P) if the interval is computed correctly (except for minor arithmetic
errors) but either the assumptions or the interpretation is not correct.

Otherwise, part (a) is scored as incorrect (I).

Part (b) is scored as either essentially correct (E) or incorrect (I). It is not possible to
score partially correct on this part.

Part (c) is scored as essentially correct (E) if all four parts of the hypothesis test are
correct. It is scored as partially correct (P) if two or three of the components of the test
are correct. Otherwise, it is scored as incorrect (I).

Part (d) is scored as essentially correct (E) if the student produces a reasonable estimate
that takes the different sample sizes into account, the explanation is correct and
communication is good.

Copyright © 2002 by College Entrance Examination Board. All rights reserved.


Advanced Placement Program and AP are registered trademarks of the College Entrance Examination Board.

24
火箭学院 火速提分
新页码 62

AP® STATISTICS
2002 SCORING GUIDELINES

Question 6 (cont’d.)

It is partially correct (P) if


• a reasonable estimate is produced but the explanation is incorrect or weak
OR
• a good explanation of why not to use the pooled estimate but no reasonable
alternative is given

For parts (a), (b), and (c), essentially correct responses count as 1 part and partially correct
responses count as ½ part.

4 Complete Response
Four parts correct.

3 Substantial Response
Three parts correct.

2 Developing Response
Two parts correct.

1 Minimal Response
One part correct.

If a paper is between two scores (for example, 2 ½ parts) use a holistic approach to determine
whether to score up or down depending on the strength of the response and communication.

Note: If the paper is between two scores and (a) or (c) has the interpretation correct, then round
up. If neither is correct, round down.

Copyright © 2002 by College Entrance Examination Board. All rights reserved.


Advanced Placement Program and AP are registered trademarks of the College Entrance Examination Board.

25
火箭学院 火速提分
新页码 63

AP® Statistics
2002 Free-Response Questions
Form B

The materials included in these files are intended for use by AP teachers for course
and exam preparation in the classroom; permission for any other use must be
sought from the Advanced Placement Program®. Teachers may reproduce them, in
whole or in part, in limited quantities, for face-to-face teaching purposes but may
not mass distribute the materials, electronically or otherwise. These materials and
any copies made of them may not be resold, and the copyright notices must be
retained as they appear here. This permission does not apply to any third-party
copyrights contained herein.

These materials were produced by Educational Testing Service® (ETS®), which develops and administers the examinations of the Advanced Placement
Program for the College Board. The College Board and Educational Testing Service (ETS) are dedicated to the principle of equal opportunity, and their
programs, services, and employment policies are guided by that principle.

The College Board is a national nonprofit membership association dedicated to preparing, inspiring, and connecting students to college and opportunity.
Founded in 1900, the association is composed of more than 4,200 schools, colleges, universities, and other educational organizations. Each year, the
College Board serves over three million students and their parents, 22,000 high schools, and 3,500 colleges, through major programs and services in
college admission, guidance, assessment, financial aid, enrollment, and teaching and learning. Among its best-known programs are the SAT®, the
PSAT/NMSQT®, and the Advanced Placement Program® (AP®). The College Board is committed to the principles of equity and
excellence, and that commitment is embodied in all of its programs, services, activities, and concerns.

Copyright © 2002 by College Entrance Examination Board. All rights reserved. College Board, Advanced Placement Program, AP, SAT, and the acorn logo
are registered trademarks of the College Entrance Examination Board. APIEL is a trademark owned by the College Entrance Examination Board. PSAT/NMSQT is a
registered trademark jointly owned by the College Entrance Examination Board and the National Merit Scholarship Corporation.
Educational Testing Service and ETS are registered trademarks of Educational Testing Service.
火箭学院 火速提分
新页码 64

2002 AP® STATISTICS FREE-RESPONSE QUESTIONS (Form B)

Formulas begin on page 3.


Questions begin on page 6.
Tables begin on page 12.

Unauthorized copying or reuse of


any part of this page is illegal.
GO ON TO THE NEXT PAGE.
2
火箭学院 火速提分
新页码 65

2002 AP® STATISTICS FREE-RESPONSE QUESTIONS (Form B)

Formulas

(I) Descriptive Statistics

∑ xi
x =
n

sx =
1
n −1
∑ xi − x
2
3 8

3n1 − 18s21 + 3n2 − 18s22


sp =
3n1 − 18 + 3n2 − 18
y$ = b0 + b1 x

b1 =
3
∑ xi − x y i − y 83 8
3
∑ xi − x 82
b0 = y − b1 x

1 x − x    yi − y 
r =
n −1
∑ i
sx    sy 
sy
b1 = r
sx

3
∑ yi − y$ i 82
sb = n−2
1
3
∑ xi − x 82

Unauthorized copying or reuse of


any part of this page is illegal.
GO ON TO THE NEXT PAGE.
3
火箭学院 火速提分
新页码 66

2002 AP® STATISTICS FREE-RESPONSE QUESTIONS (Form B)

(II) Probability

P( A ∪ B) = P( A) + P( B) − P( A ∩ B)

P( A ∩ B )
P( A B ) =
P( B )

E ( X ) = µ x = ∑ xi pi

3
Var( X ) = s 2x = ∑ xi − µ x 82 pi

If X has a binomial distribution with parameters n and p, then:

 n p k (1 − p)n − k
P( X = k ) =
 k
µ x = np

s x = np(1 − p)

µ$ = p
p

p(1 − p)
s p$ =
n

If x is the mean of a random sample of size n from an infinite


population with mean µ and standard deviation s , then:

µx = µ

sx = s
n

Unauthorized copying or reuse of


any part of this page is illegal.
GO ON TO THE NEXT PAGE.
4
火箭学院 火速提分
新页码 67

2002 AP® STATISTICS FREE-RESPONSE QUESTIONS (Form B)

(III) Inferential Statistics

statistic - parameter
Standardized test statistic:
standard deviation of statistic

Confidence interval: statistic – (critical value ) œ (standard deviation of statistic)

Single-Sample

Standard Deviation
Statistic
of Statistic
σ
Sample Mean n

p(1 − p)
Sample Proportion n

Two-Sample

Standard Deviation
Statistic
of Statistic

Difference of σ 12 σ 22
sample means +
n1 n2

Special case when σ 1 = σ 2

1 1
σ +
n1 n2

Difference of p1 (1 − p1 ) p (1 − p2 )
sample proportions + 2
n1 n2

Special case when p1 = p2

1 6
p 1− p
1
+
1
n1 n2

Chi-square test statistic = ∑


0observed − expected5 2

expected

Unauthorized copying or reuse of


any part of this page is illegal.
GO ON TO THE NEXT PAGE.
5
火箭学院 火速提分
新页码 68

2002 AP® STATISTICS FREE-RESPONSE QUESTIONS (Form B)

STATISTICS
SECTION II
Part A
Questions 1-5
Spend about 65 minutes on this part of the exam.
Percent of Section II grade—75

Directions: Show all your work. Indicate clearly the methods you use, because you will be graded on the
correctness of your methods as well as on the accuracy of your results and explanation.

1. Animal-waste lagoons and spray fields near aquatic environments may significantly degrade water quality and
endanger health. The National Atmospheric Deposition Program has monitored the atmospheric ammonia at
swine farms since 1978. The data on the swine population size (in thousands) and atmospheric ammonia (in parts
per million) for one decade are given below.

Year 1988 1989 1990 1991 1992 1993 1994 1995 1996 1997
Swine 0.38 0.50 0.60 0.75 0.95 1.20 1.40 1.65 1.80 1.85
Population
Atmospheric 0.13 0.21 0.29 0.22 0.19 0.26 0.36 0.37 0.33 0.38
Ammonia

(a) Construct a scatterplot for these data.

Copyright © 2002 by College Entrance Examination Board. All rights reserved.


Advanced Placement Program and AP are registered trademarks of the College Entrance Examination Board.

GO ON TO THE NEXT PAGE.


6
火箭学院 火速提分
新页码 69

2002 AP® STATISTICS FREE-RESPONSE QUESTIONS (Form B)

(b) The value for the correlation coefficient for these data is 0.85. Interpret this value.
(c) Based on the scatterplot in part (a) and the value of the correlation coefficient in part (b), does it appear that
the amount of atmospheric ammonia is linearly related to the swine population size?
Explain.
(d) What percent of the variability in atmospheric ammonia can be explained by swine population size?

2. Airlines routinely overbook flights because they expect a certain number of no-shows. An airline runs a 5 P.M.
commuter flight from Washington, D.C., to New York City on a plane that holds 38 passengers. Past experience
has shown that if 41 tickets are sold for the flight, then the probability distribution for the number who actually
show up for the flight is as shown in the table below.

Number who 36 37 38 39 40 41
actually show up
Probability 0.46 0.30 0.16 0.05 0.02 0.01

Assume that 41 tickets are sold for each flight.


(a) There are 38 passenger seats on the flight. What is the probability that all passengers who show up for this
flight will get a seat?
(b) What is the expected number of no-shows for this flight?
(c) Given that not all passenger seats are filled on a flight, what is the probability that only 36 passengers
showed up for the flight?

3. A preliminary study conducted at a medical center in St. Louis has shown that treatment with small, low-
intensity magnets reduces the self-reported level of pain in polio patients. During each session, a patient rested
on an examining table in the doctor’s office while the magnets, embedded in soft pads, were strapped to the body
at the site of pain. Sessions continued for several weeks, after which pain reduction was measured.
A new study is being designed to investigate whether magnets also reduce pain in patients suffering from
herniated disks in the lower back. One hundred male patients are available for the new study.
(a) Describe an appropriate design for the new study. Your discussion should briefly address treatments used,
methods of treatment assignment, and what variables would be measured. Do not describe how the data
would be analyzed.
(b) Would you modify the design above if, instead of 100 male patients, there were 50 male and 50 female
patients available for the study? If so, how would you modify your design? If not, why not?

Copyright © 2002 by College Entrance Examination Board. All rights reserved.


Advanced Placement Program and AP are registered trademarks of the College Entrance Examination Board.

GO ON TO THE NEXT PAGE.


7
火箭学院 火速提分
新页码 70

2002 AP® STATISTICS FREE-RESPONSE QUESTIONS (Form B)

4. Each person in a random sample of 1,026 adults in the United States was asked the following question.
“Based on what you know about the Social Security system today, what would you like Congress and the
President to do during this next year?”
The response choices and the percentages selecting them are shown below.

Completely overhaul the system 19%


Make some major changes 39%
Make some minor adjustments 30%
Leave the system the way it is now 11%
No opinion 1%

(a) Find a 95% confidence interval for the proportion of all United States adults who would respond “Make
some major changes” to the question. Give an interpretation of the confidence interval and give an
interpretation of the confidence level.

(b) An advocate for leaving the system as it is now commented, “Based on this poll, only 39% of adults in the
sample responded that they want some major changes made to the system, while 41% responded that they
want only minor changes or no changes at all. Therefore, we should not change the system.” Explain why
this statement, while technically correct, is misleading.

Copyright © 2002 by College Entrance Examination Board. All rights reserved.


Advanced Placement Program and AP are registered trademarks of the College Entrance Examination Board.

GO ON TO THE NEXT PAGE.


8
火箭学院 火速提分
新页码 71

2002 AP® STATISTICS FREE-RESPONSE QUESTIONS (Form B)

5. At a school field day, 50 students and 50 faculty members each completed an obstacle course. Descriptive
statistics for the completion times (in minutes) for the two groups are shown below.

Students Faculty Members


Mean 9.90 12.09
Median 9.25 11.00
Minimum 3.75 4.50
Maximum 16.50 25.00
Lower quartile 6.75 8.75
Upper quartile 13.75 15.75

(a) Use the same scale to draw boxplots for the completion times for students and for faculty members.
(b) Write a few sentences comparing the variability of the two distributions.
(c) You have been asked to report on this event for the school newspaper. Write a few sentences describing
student and faculty performances in this competition for the paper.

Copyright © 2002 by College Entrance Examination Board. All rights reserved.


Advanced Placement Program and AP are registered trademarks of the College Entrance Examination Board.

GO ON TO THE NEXT PAGE.


9
火箭学院 火速提分
新页码 72

2002 AP® STATISTICS FREE-RESPONSE QUESTIONS (Form B)

STATISTICS
SECTION II
Part B
Question 6
Spend about 25 minutes on this part of the exam.
Percent of Section II grade—25

Directions: Show all your work. Indicate clearly the methods you use, because you will be graded on the
correctness of your methods as well as on the accuracy of your results and explanation.

6. In September 1990, each student in a random sample of 200 biology majors at a large university was asked how
many lab classes he or she was enrolled in. The sample results are shown below.

Number of Lab Classes Number of Students


0 28
1 62
2 58 x = 1.83
3 28 s = 1.29
4 16
5 8
(Total) 200

To determine whether the distribution has changed over the past 10 years, a similar survey was conducted in
September 2000 by selecting a random sample of 200 biology majors. Results from the year 2000 sample are
shown below.

Number of Lab Classes Number of Students


0 20
1 72
2 60 x = 1.93
3 10 s = 1.37
4 26
5 12
(Total) 200

(a) Do the data provide evidence that the mean number of lab classes taken by biology majors in September
2000 was different from the mean number of lab classes taken in 1990 ? Perform an appropriate statistical
test using α = 010
. to answer this question.
(b) Does the test in (a) address the question of whether the distribution of number of lab classes was different in
2000 than it was in 1990 ? If so, explain your reasoning. If not, carry out an appropriate statistical test using
α = 0.10 to answer this question.

Copyright © 2002 by College Entrance Examination Board. All rights reserved.


Advanced Placement Program and AP are registered trademarks of the College Entrance Examination Board.

GO ON TO THE NEXT PAGE.


10
火箭学院 火速提分
新页码 73

2002 AP® STATISTICS FREE-RESPONSE QUESTIONS (Form B)

(c) Use the results of your analyses in (a) and (b) to write a few sentences that summarize how the distribution
of the number of lab classes did or did not differ. Use appropriate graphs to help communicate your
message. This summary should be understandable to someone who has not studied statistics.

END OF EXAMINATION

Copyright © 2002 by College Entrance Examination Board. All rights reserved.


Advanced Placement Program and AP are registered trademarks of the College Entrance Examination Board.

GO ON TO THE NEXT PAGE.


11
火箭学院 火速提分
新页码 74

2002 AP® STATISTICS FREE-RESPONSE QUESTIONS (Form B)

Probability

Table entry for z is the


probability lying below z.
z

Table A Standard normal probabilities

z .00 .01 .02 .03 .04 .05 .06 .07 .08 .09
– 3.4 .0003 .0003 .0003 .0003 .0003 .0003 .0003 .0003 .0003 .0002
– 3.3 .0005 .0005 .0005 .0004 .0004 .0004 .0004 .0004 .0004 .0003
– 3.2 .0007 .0007 .0006 .0006 .0006 .0006 .0006 .0005 .0005 .0005
– 3.1 .0010 .0009 .0009 .0009 .0008 .0008 .0008 .0008 .0007 .0007
– 3.0 .0013 .0013 .0013 .0012 .0012 .0011 .0011 .0011 .0010 .0010
– 2.9 .0019 .0018 .0018 .0017 .0016 .0016 .0015 .0015 .0014 .0014
– 2.8 .0026 .0025 .0024 .0023 .0023 .0022 .0021 .0021 .0020 .0019
– 2.7 .0035 .0034 .0033 .0032 .0031 .0030 .0029 .0028 .0027 .0026
– 2.6 .0047 .0045 .0044 .0043 .0041 .0040 .0039 .0038 .0037 .0036
– 2.5 .0062 .0060 .0059 .0057 .0055 .0054 .0052 .0051 .0049 .0048
– 2.4 .0082 .0080 .0078 .0075 .0073 .0071 .0069 .0068 .0066 .0064
– 2.3 .0107 .0104 .0102 .0099 .0096 .0094 .0091 .0089 .0087 .0084
– 2.2 .0139 .0136 .0132 .0129 .0125 .0122 .0119 .0116 .0113 .0110
– 2.1 .0179 .0174 .0170 .0166 .0162 .0158 .0154 .0150 .0146 .0143
– 2.0 .0228 .0222 .0217 .0212 .0207 .0202 .0197 .0192 .0188 .0183
– 1.9 .0287 .0281 .0274 .0268 .0262 .0256 .0250 .0244 .0239 .0233
– 1.8 .0359 .0351 .0344 .0336 .0329 .0322 .0314 .0307 .0301 .0294
– 1.7 .0446 .0436 .0427 .0418 .0409 .0401 .0392 .0384 .0375 .0367
– 1.6 .0548 .0537 .0526 .0516 .0505 .0495 .0485 .0475 .0465 .0455
– 1.5 .0668 .0655 .0643 .0630 .0618 .0606 .0594 .0582 .0571 .0559
– 1.4 .0808 .0793 .0778 .0764 .0749 .0735 .0721 .0708 .0694 .0681
– 1.3 .0968 .0951 .0934 .0918 .0901 .0885 .0869 .0853 .0838 .0823
– 1.2 .1151 .1131 .1112 .1093 .1075 .1056 .1038 .1020 .1003 .0985
– 1.1 .1357 .1335 .1314 .1292 .1271 .1251 .1230 .1210 .1190 .1170
– 1.0 .1587 .1562 .1539 .1515 .1492 .1469 .1446 .1423 .1401 .1379
– 0.9 .1841 .1814 .1788 .1762 .1736 .1711 .1685 .1660 .1635 .1611
– 0.8 .2119 .2090 .2061 .2033 .2005 .1977 .1949 .1922 .1894 .1867
– 0.7 .2420 .2389 .2358 .2327 .2296 .2266 .2236 .2206 .2177 .2148
– 0.6 .2743 .2709 .2676 .2643 .2611 .2578 .2546 .2514 .2483 .2451
– 0.5 .3085 .3050 .3015 .2981 .2946 .2912 .2877 .2843 .2810 .2776
– 0.4 .3446 .3409 .3372 .3336 .3300 .3264 .3228 .3192 .3156 .3121
– 0.3 .3821 .3783 .3745 .3707 .3669 .3632 .3594 .3557 .3520 .3483
– 0.2 .4207 .4168 .4129 .4090 .4052 .4013 .3974 .3936 .3897 .3859
– 0.1 .4602 .4562 .4522 .4483 .4443 .4404 .4364 .4325 .4286 .4247
– 0.0 .5000 .4960 .4920 .4880 .4840 .4801 .4761 .4721 .4681 .4641

12
火箭学院 火速提分
新页码 75

2002 AP® STATISTICS FREE-RESPONSE QUESTIONS (Form B)

Probability

Table entry for z is the


probability lying below z.
z
Table A (Continued)

z .00 .01 .02 .03 .04 .05 .06 .07 .08 .09
0.0 .5000 .5040 .5080 .5120 .5160 .5199 .5239 .5279 .5319 .5359
0.1 .5398 .5438 .5478 .5517 .5557 .5596 .5636 .5675 .5714 .5753
0.2 .5793 .5832 .5871 .5910 .5948 .5987 .6026 .6064 .6103 .6141
0.3 .6179 .6217 .6255 .6293 .6331 .6368 .6406 .6443 .6480 .6517
0.4 .6554 .6591 .6628 .6664 .6700 .6736 .6772 .6808 .6844 .6879
0.5 .6915 .6950 .6985 .7019 .7054 .7088 .7123 .7157 .7190 .7224
0.6 .7257 .7291 .7324 .7357 .7389 .7422 .7454 .7486 .7517 .7549
0.7 .7580 .7611 .7642 .7673 .7704 .7734 .7764 .7794 .7823 .7852
0.8 .7881 .7910 .7939 .7967 .7995 .8023 .8051 .8078 .8106 .8133
0.9 .8159 .8186 .8212 .8238 .8264 .8289 .8315 .8340 .8365 .8389
1.0 .8413 .8438 .8461 .8485 .8508 .8531 .8554 .8577 .8599 .8621
1.1 .8643 .8665 .8686 .8708 .8729 .8749 .8770 .8790 .8810 .8830
1.2 .8849 .8869 .8888 .8907 .8925 .8944 .8962 .8980 .8997 .9015
1.3 .9032 .9049 .9066 .9082 .9099 .9115 .9131 .9147 .9162 .9177
1.4 .9192 .9207 .9222 .9236 .9251 .9265 .9279 .9292 .9306 .9319
1.5 .9332 .9345 .9357 .9370 .9382 .9394 .9406 .9418 .9429 .9441
1.6 .9452 .9463 .9474 .9484 .9495 .9505 .9515 .9525 .9535 .9545
1.7 .9554 .9564 .9573 .9582 .9591 .9599 .9608 .9616 .9625 .9633
1.8 .9641 .9649 .9656 .9664 .9671 .9678 .9686 .9693 .9699 .9706
1.9 .9713 .9719 .9726 .9732 .9738 .9744 .9750 .9756 .9761 .9767
2.0 .9772 .9778 .9783 .9788 .9793 .9798 .9803 .9808 .9812 .9817
2.1 .9821 .9826 .9830 .9834 .9838 .9842 .9846 .9850 .9854 .9857
2.2 .9861 .9864 .9868 .9871 .9875 .9878 .9881 .9884 .9887 .9890
2.3 .9893 .9896 .9898 .9901 .9904 .9906 .9909 .9911 .9913 .9916
2.4 .9918 .9920 .9922 .9925 .9927 .9929 .9931 .9932 .9934 .9936
2.5 .9938 .9940 .9941 .9943 .9945 .9946 .9948 .9949 .9951 .9952
2.6 .9953 .9955 .9956 .9957 .9959 .9960 .9961 .9962 .9963 .9964
2.7 .9965 .9966 .9967 .9968 .9969 .9970 .9971 .9972 .9973 .9974
2.8 .9974 .9975 .9976 .9977 .9977 .9978 .9979 .9979 .9980 .9981
2.9 .9981 .9982 .9982 .9983 .9984 .9984 .9985 .9985 .9986 .9986
3.0 .9987 .9987 .9987 .9988 .9988 .9989 .9989 .9989 .9990 .9990
3.1 .9990 .9991 .9991 .9991 .9992 .9992 .9992 .9992 .9993 .9993
3.2 .9993 .9993 .9994 .9994 .9994 .9994 .9994 .9995 .9995 .9995
3.3 .9995 .9995 .9995 .9996 .9996 .9996 .9996 .9996 .9996 .9997
3.4 .9997 .9997 .9997 .9997 .9997 .9997 .9997 .9997 .9997 .9998

13
火箭学院 火速提分
新页码 76

2002 AP® STATISTICS FREE-RESPONSE QUESTIONS (Form B)

Table entry for p and


C is the point t* with Probability p
probability p lying
above it and
probability C lying
between −t * and t*.

t*

Table B t distribution critical values

Tail probability p
df .25 .20 .15 .10 .05 .025 .02 .01 .005 .0025 .001 .0005
1 1.000 1.376 1.963 3.078 6.314 12.71 15.89 31.82 63.66 127.3 318.3 636.6
2 .816 1.061 1.386 1.886 2.920 4.303 4.849 6.965 9.925 14.09 22.33 31.60
3 .765 .978 1.250 1.638 2.353 3.182 3.482 4.541 5.841 7.453 10.21 12.92
4 .741 .941 1.190 1.533 2.132 2.776 2.999 3.747 4.604 5.598 7.173 8.610
5 .727 .920 1.156 1.476 2.015 2.571 2.757 3.365 4.032 4.773 5.893 6.869
6 .718 .906 1.134 1.440 1.943 2.447 2.612 3.143 3.707 4.317 5.208 5.959
7 .711 .896 1.119 1.415 1.895 2.365 2.517 2.998 3.499 4.029 4.785 5.408
8 .706 .889 1.108 1.397 1.860 2.306 2.449 2.896 3.355 3.833 4.501 5.041
9 .703 .883 1.100 1.383 1.833 2.262 2.398 2.821 3.250 3.690 4.297 4.781
10 .700 .879 1.093 1.372 1.812 2.228 2.359 2.764 3.169 3.581 4.144 4.587
11 .697 .876 1.088 1.363 1.796 2.201 2.328 2.718 3.106 3.497 4.025 4.437
12 .695 .873 1.083 1.356 1.782 2.179 2.303 2.681 3.055 3.428 3.930 4.318
13 .694 .870 1.079 1.350 1.771 2.160 2.282 2.650 3.012 3.372 3.852 4.221
14 .692 .868 1.076 1.345 1.761 2.145 2.264 2.624 2.977 3.326 3.787 4.140
15 .691 .866 1.074 1.341 1.753 2.131 2.249 2.602 2.947 3.286 3.733 4.073
16 .690 .865 1.071 1.337 1.746 2.120 2.235 2.583 2.921 3.252 3.686 4.015
17 .689 .863 1.069 1.333 1.740 2.110 2.224 2.567 2.898 3.222 3.646 3.965
18 .688 .862 1.067 1.330 1.734 2.101 2.214 2.552 2.878 3.197 3.611 3.922
19 .688 .861 1.066 1.328 1.729 2.093 2.205 2.539 2.861 3.174 3.579 3.883
20 .687 .860 1.064 1.325 1.725 2.086 2.197 2.528 2.845 3.153 3.552 3.850
21 .686 .859 1.063 1.323 1.721 2.080 2.189 2.518 2.831 3.135 3.527 3.819
22 .686 .858 1.061 1.321 1.717 2.074 2.183 2.508 2.819 3.119 3.505 3.792
23 .685 .858 1.060 1.319 1.714 2.069 2.177 2.500 2.807 3.104 3.485 3.768
24 .685 .857 1.059 1.318 1.711 2.064 2.172 2.492 2.797 3.091 3.467 3.745
25 .684 .856 1.058 1.316 1.708 2.060 2.167 2.485 2.787 3.078 3.450 3.725
26 .684 .856 1.058 1.315 1.706 2.056 2.162 2.479 2.779 3.067 3.435 3.707
27 .684 .855 1.057 1.314 1.703 2.052 2.158 2.473 2.771 3.057 3.421 3.690
28 .683 .855 1.056 1.313 1.701 2.048 2.154 2.467 2.763 3.047 3.408 3.674
29 .683 .854 1.055 1.311 1.699 2.045 2.150 2.462 2.756 3.038 3.396 3.659
30 .683 .854 1.055 1.310 1.697 2.042 2.147 2.457 2.750 3.030 3.385 3.646
40 .681 .851 1.050 1.303 1.684 2.021 2.123 2.423 2.704 2.971 3.307 3.551
50 .679 .849 1.047 1.299 1.676 2.009 2.109 2.403 2.678 2.937 3.261 3.496
60 .679 .848 1.045 1.296 1.671 2.000 2.099 2.390 2.660 2.915 3.232 3.460
80 .678 .846 1.043 1.292 1.664 1.990 2.088 2.374 2.639 2.887 3.195 3.416
100 .677 .845 1.042 1.290 1.660 1.984 2.081 2.364 2.626 2.871 3.174 3.390
1000 .675 .842 1.037 1.282 1.646 1.962 2.056 2.330 2.581 2.813 3.098 3.300
⬁ .674 .841 1.036 1.282 1.645 1.960 2.054 2.326 2.576 2.807 3.091 3.291

50% 60% 70% 80% 90% 95% 96% 98% 99% 99.5% 99.8% 99.9%
Confidence level C

14
火箭学院 火速提分
新页码 77

2002 AP® STATISTICS FREE-RESPONSE QUESTIONS (Form B)

Probability p
Table entry for p is the point
( χ 2 ) with probability p lying
above it.

(χ2 )

Table C χ 2 critical values


Tail probability p
df .25 .20 .15 .10 .05 .025 .02 .01 .005 .0025 .001 .0005
1 1.32 1.64 2.07 2.71 3.84 5.02 5.41 6.63 7.88 9.14 10.83 12.12
2 2.77 3.22 3.79 4.61 5.99 7.38 7.82 9.21 10.60 11.98 13.82 15.20
3 4.11 4.64 5.32 6.25 7.81 9.35 9.84 11.34 12.84 14.32 16.27 17.73
4 5.39 5.99 6.74 7.78 9.49 11.14 11.67 13.28 14.86 16.42 18.47 20.00
5 6.63 7.29 8.12 9.24 11.07 12.83 13.39 15.09 16.75 18.39 20.51 22.11
6 7.84 8.56 9.45 10.64 12.59 14.45 15.03 16.81 18.55 20.25 22.46 24.10
7 9.04 9.80 10.75 12.02 14.07 16.01 16.62 18.48 20.28 22.04 24.32 26.02
8 10.22 11.03 12.03 13.36 15.51 17.53 18.17 20.09 21.95 23.77 26.12 27.87
9 11.39 12.24 13.29 14.68 16.92 19.02 19.68 21.67 23.59 25.46 27.88 29.67
10 12.55 13.44 14.53 15.99 18.31 20.48 21.16 23.21 25.19 27.11 29.59 31.42
11 13.70 14.63 15.77 17.28 19.68 21.92 22.62 24.72 26.76 28.73 31.26 33.14
12 14.85 15.81 16.99 18.55 21.03 23.34 24.05 26.22 28.30 30.32 32.91 34.82
13 15.98 16.98 18.20 19.81 22.36 24.74 25.47 27.69 29.82 31.88 34.53 36.48
14 17.12 18.15 19.41 21.06 23.68 26.12 26.87 29.14 31.32 33.43 36.12 38.11
15 18.25 19.31 20.60 22.31 25.00 27.49 28.26 30.58 32.80 34.95 37.70 39.72
16 19.37 20.47 21.79 23.54 26.30 28.85 29.63 32.00 34.27 36.46 39.25 41.31
17 20.49 21.61 22.98 24.77 27.59 30.19 31.00 33.41 35.72 37.95 40.79 42.88
18 21.60 22.76 24.16 25.99 28.87 31.53 32.35 34.81 37.16 39.42 42.31 44.43
19 22.72 23.90 25.33 27.20 30.14 32.85 33.69 36.19 38.58 40.88 43.82 45.97
20 23.83 25.04 26.50 28.41 31.41 34.17 35.02 37.57 40.00 42.34 45.31 47.50
21 24.93 26.17 27.66 29.62 32.67 35.48 36.34 38.93 41.40 43.78 46.80 49.01
22 26.04 27.30 28.82 30.81 33.92 36.78 37.66 40.29 42.80 45.20 48.27 50.51
23 27.14 28.43 29.98 32.01 35.17 38.08 38.97 41.64 44.18 46.62 49.73 52.00
24 28.24 29.55 31.13 33.20 36.42 39.36 40.27 42.98 45.56 48.03 51.18 53.48
25 29.34 30.68 32.28 34.38 37.65 40.65 41.57 44.31 46.93 49.44 52.62 54.95
26 30.43 31.79 33.43 35.56 38.89 41.92 42.86 45.64 48.29 50.83 54.05 56.41
27 31.53 32.91 34.57 36.74 40.11 43.19 44.14 46.96 49.64 52.22 55.48 57.86
28 32.62 34.03 35.71 37.92 41.34 44.46 45.42 48.28 50.99 53.59 56.89 59.30
29 33.71 35.14 36.85 39.09 42.56 45.72 46.69 49.59 52.34 54.97 58.30 60.73
30 34.80 36.25 37.99 40.26 43.77 46.98 47.96 50.89 53.67 56.33 59.70 62.16
40 45.62 47.27 49.24 51.81 55.76 59.34 60.44 63.69 66.77 69.70 73.40 76.09
50 56.33 58.16 60.35 63.17 67.50 71.42 72.61 76.15 79.49 82.66 86.66 89.56
60 66.98 68.97 71.34 74.40 79.08 83.30 84.58 88.38 91.95 95.34 99.61 102.7
80 88.13 90.41 93.11 96.58 101.9 106.6 108.1 112.3 116.3 120.1 124.8 128.3
100 109.1 111.7 114.7 118.5 124.3 129.6 131.1 135.8 140.2 144.3 149.4 153.2

15
火箭学院 火速提分
新页码 78

AP® Statistics
2002 Scoring Guidelines
Form B

The materials included in these files are intended for use by AP teachers for course
and exam preparation in the classroom; permission for any other use must be
sought from the Advanced Placement Program®. Teachers may reproduce them, in
whole or in part, in limited quantities, for face-to-face teaching purposes but may
not mass distribute the materials, electronically or otherwise. These materials and
any copies made of them may not be resold, and the copyright notices must be
retained as they appear here. This permission does not apply to any third-party
copyrights contained herein.

These materials were produced by Educational Testing Service® (ETS®), which develops and administers the examinations of the Advanced Placement
Program for the College Board. The College Board and Educational Testing Service (ETS) are dedicated to the principle of equal opportunity, and their
programs, services, and employment policies are guided by that principle.

The College Board is a national nonprofit membership association dedicated to preparing, inspiring, and connecting students to college and opportunity.
Founded in 1900, the association is composed of more than 4,200 schools, colleges, universities, and other educational organizations. Each year, the
College Board serves over three million students and their parents, 22,000 high schools, and 3,500 colleges, through major programs and services in
college admission, guidance, assessment, financial aid, enrollment, and teaching and learning. Among its best-known programs are the SAT®, the
PSAT/NMSQT®, and the Advanced Placement Program® (AP®). The College Board is committed to the principles of equity and
excellence, and that commitment is embodied in all of its programs, services, activities, and concerns.

Copyright © 2002 by College Entrance Examination Board. All rights reserved. College Board, Advanced Placement Program, AP, SAT, and the acorn logo
are registered trademarks of the College Entrance Examination Board. APIEL is a trademark owned by the College Entrance Examination Board. PSAT/NMSQT is a
registered trademark jointly owned by the College Entrance Examination Board and the National Merit Scholarship Corporation.
Educational Testing Service and ETS are registered trademarks of Educational Testing Service.
火箭学院 火速提分
新页码 79

AP® STATISTICS
2002 SCORING GUIDELINES (Form B)

Question 1

Solution

Part (a):

0
1988 1989 1990 1991 1992 1993 1994 1995 1996 1997

Part (b):
There is a strong, positive, linear relationship between swine population size and
atmospheric ammonia.

Part (c):
Both the value of the correlation coefficient and the pattern in the scatterplot indicate that
there is a positive linear relationship between the size of the swine population and
atmospheric ammonia.

Part (d):
r2 = .72 or 72% or 72
Note: Just writing r2 is not sufficient.

Copyright © 2002 by College Entrance Examination Board. All rights reserved.


Advanced Placement Program and AP are registered trademarks of the College Entrance Examination Board.

2
火箭学院 火速提分
新页码 80

AP® STATISTICS
2002 SCORING GUIDELINES (Form B)

Question 1 (cont’d.)

Scoring

Part (a) is considered

Essentially correct if a scatterplot is given that includes axis labels and scales.

Partially correct if a scatterplot is given but only one of (i) the axis labels or (ii) the
scales is included.

Incorrect if neither labels or scales are included.

Parts (b) and (c) should be read “collectively.”

Part (b) is considered

Essentially correct if the interpretation of the correlation coefficient includes both strong
and positive, and is interpreted in context.

Partially correct if the interpretation mentions two of the components: strong, positive,
and context.

Parts (c) is considered

Essentially correct if comments are correct and are based on both the value of the
correlation coefficient and the scatterplot.

Partially correct if it includes a correct comment based on only one of the components:
correlation coefficient and the scatterplot.

Part (d) is considered

Essentially correct or incorrect.

NOTE: In order to recoup strength or direction for part (b) in part (c), the comments must be
directly tied to the correlation coefficient. For example, saying the variables increase
together does not count as saying the correlation tells us the association is positive.
Interpretations in context in (c) can recoup missing context in (b).

NOTE: A construction such as “as x increases, y increases” counts as indicating a positive


association, but does not by itself count as indicating linearity.

NOTE: A statement such as “judging by the scatterplot and correlation” without appealing to some
characteristic of the plot or that the correlation is .85, is scored as partially correct.

Essentially correct responses count as 1 part and partially correct responses count as ½ part. If a
paper is between two scores (for example 2 ½ parts), use a holistic approach to determine whether
to score up or down depending on the strength of the response and communication.
Copyright © 2002 by College Entrance Examination Board. All rights reserved.
Advanced Placement Program and AP are registered trademarks of the College Entrance Examination Board.

3
火箭学院 火速提分
新页码 81

AP® STATISTICS
2002 SCORING GUIDELINES (Form B)

Question 1 (cont’d.)

4 Complete Response
Four parts correct.

3 Substantial Response
Three parts correct.

2 Developing Response
Two parts correct.

1 Minimal Response
One part correct.

Copyright © 2002 by College Entrance Examination Board. All rights reserved.


Advanced Placement Program and AP are registered trademarks of the College Entrance Examination Board.

4
火箭学院 火速提分
新页码 82

AP® STATISTICS
2002 SCORING GUIDELINES (Form B)

Question 2

Solution

Part (a):

P (everyone gets a seat) = P (X ≤ 38) = .46 + .30 + .16 = .92

OR = 1 – (.05 + .02 + .01) = .92

Part (b):

Y = number of no shows

y 0 1 2 3 4 5
p(y) .01 .02 .05 .16 .30 .46

E(Y) = 0(.01) + 1(.02) + 2(.05) + 3(.16) + 4(.30) + 5(.46) = 4.1

OR

E(X) = 36(.46) + 37(.30) + 38(.16) + 39(.05) + 40(.02) + 41(.01) = 36.9

E(Y) = 41 – E(X) = 4.1

Part (c):

P( X = 36) .46
P(X=36|not all seats are filled) = P ( X = 36 | X < 38) = = = .605
P( X < 38) .76

Copyright © 2002 by College Entrance Examination Board. All rights reserved.


Advanced Placement Program and AP are registered trademarks of the College Entrance Examination Board.

5
火箭学院 火速提分
新页码 83

AP® STATISTICS
2002 SCORING GUIDELINES (Form B)

Question 2 (cont’d.)

Scoring

Part (a) is

Essentially correct if computes P(X ≤ 38) (except for minor arithmetic errors)

Partially correct if computes P(X = 38) = 0.16 or P(X > 38) = .08 or P(X < 38) = 0.76 or
P ( X ³ 38) = 0.24 or gives 0.92 but does not show any work.

38 36.9
Incorrect if no calculation or nonsensical calculation (e.g., 0.921 , = 0.927 , = 0.971 ) or
41 38
only pure expected value calculations.

Part (b) is

Essentially correct if (except for minor arithmetic errors) correctly computes expected value for
number of no shows and indicates fully where the 36.9 and 4.1 come from.

Rounding this value to 4 is considered a minus, though can be forgiven with “about” or
“approximately.”

Partially correct if correctly computes expected value of X = number of


passengers who show up for the flight (instead of no shows) and shows work,

OR if incorrectly computes the first expected value but “subtracts,”

OR if correctly computes the first expected value but “subtracts” from the wrong number,

OR if does not show work for 36.9 but subtracts to get 4.1.

Incorrect if is not an expected value or does not use all six outcomes in the expected value.

Part (c) is

Essentially correct if correctly computes the conditional probability. Complete notation could
be considered a “plus.”

0.46
Partially correct if correctly computes P ( X = 36 | X < 41) = = 0.465 or
0.99
0.46
P ( X = 36 | X £ 38) = = 0.5 or incorrectly tries to solve the conditional probability (e.g.,
0.92
multiplies probabilities in numerator).

Incorrect if an unconditional probability is computed.

Copyright © 2002 by College Entrance Examination Board. All rights reserved.


Advanced Placement Program and AP are registered trademarks of the College Entrance Examination Board.

6
火箭学院 火速提分
新页码 84

AP® STATISTICS
2002 SCORING GUIDELINES (Form B)

Question 2 (cont’d.)

4 Complete Response

Essentially correct on all three parts.

3 Substantial Response

Essentially correct on two parts and partially correct on the other part or incorrect on (a)
and essentially correct on (b) and (c).

2 Developing Response

Essentially correct on two parts and incorrect on the other (except IEE – see above).
OR
Essentially correct on one part and partially correct on the other two parts.
OR
Partially correct on all three parts.
OR
Essentially correct on one part, partial on another, and incorrect on a third part.

1 Minimal Response

Essentially correct on one part and in correct on the other two parts.
OR
Partially correct on one or two parts and incorrect on the other.
PII should be graded holistically (needs something elsewhere for a 1).

Copyright © 2002 by College Entrance Examination Board. All rights reserved.


Advanced Placement Program and AP are registered trademarks of the College Entrance Examination Board.

7
火箭学院 火速提分
新页码 85

AP® STATISTICS
2002 SCORING GUIDELINES (Form B)

Question 3

Solution

Part (a):
1. Two treatments: magnets and no magnets (or magnets and placebo). Subjects in
the no magnet group would be handled in the same way as the magnet group, but
there would be no magnets embedded in the pads used.
2. There must be random assignment of subjects to treatments (or treatments to
subjects). How the randomization would be carried out does not need to be
specified, but it must be clear what is being randomized.
3. Variable measured: Self-reported level of pain or reduction in pain.

The design may be described by a diagram, but the treatments and the variable measured
must be included and the randomization must be very clear.

Group 1 Magnets
Measure and
Random compare self-
assignment reported level of
of subjects
pain
Group 2 No Magnets
(or Placebo)

Part (b): Either one of the following approaches is acceptable.


1. Saying yes and indicating how they would alter the design: Separating the subjects
into the two gender groups and then randomizing subjects to treatments within each
group. This may also be described using a diagram, as shown below, but the
blocking factor and randomization must be clearly indicated.

OR

2. Saying no and describing why. For example, indicating that the randomization in (a)
should equalize the effects of gender in the two groups or assuming gender does not
have a strong effect and since the sample size is large
OR
providing a good explanation for why gender does not have a strong differential
effect on the outcome.

Copyright © 2002 by College Entrance Examination Board. All rights reserved.


Advanced Placement Program and AP are registered trademarks of the College Entrance Examination Board.

8
火箭学院 火速提分
新页码 86

AP® STATISTICS
2002 SCORING GUIDELINES (Form B)

Question 3 (cont’d.)

Magnets
Random Measure and
Block 1 assignment compare
(Female) of females No Magnets self-reported
Separate (or Placebo) level of pain
subjects by
gender into
Block 2 Random Magnets Measure and
2 blocks by
(Male) assignment compare
of males No Magnets self-reported
(or Placebo) level of pain

Scoring

Part (a) is

Essentially correct if all three of the following are adequately addressed:


treatments, random assignment, and response variable.

It is acceptable for them to select the first group with an SRS, but only if it’s
clear that the remaining subjects are automatically assigned to the second
treatment group as opposed to taking a separate SRS.

Note, if they describe a randomization scheme that results in an unequal number


of subjects in each treatment group or does not involve randomization for each
individual (such as flipping a coin to determine which group gets the placebo),
this can be considered a minus.

The student may also describe the experimental protocol in more detail – double
blind, placebo effect, etc. These are not necessary but can be considered a plus.

Partially correct if only one or two of the items listed above are adequately
addressed. But, to receive the partial with only one item, the design must be
described exceptionally well. For example, discussion of the placebo effect as
reason for the control group, or a “before and after” measurement for the
response, or a detailed discussion of randomization. (A well done “before and
after” design in the new context can be considered partially correct.)

NOTE: Students do not get credit for discussing treatments if their design does
not ever specify a control group or fails to define the treatment.

Copyright © 2002 by College Entrance Examination Board. All rights reserved.


Advanced Placement Program and AP are registered trademarks of the College Entrance Examination Board.

9
火箭学院 火速提分
新页码 87

AP® STATISTICS
2002 SCORING GUIDELINES (Form B)

Question 3 (cont’d.)

Part (b) is

Essentially correct if either of the two acceptable approaches is taken (no and why OR
yes and how) and the explanation/justification is clear and complete. For a yes,
the response must clearly indicate and maintain the group separation and specify
the separate randomization within each group.

Partially correct if either of the two acceptable approaches is taken, but the
explanation/justification is weak. For example, the response states yes but it is
not clear a separate randomization within each block is carried out, or the
response states no but does not fully appeal to the randomization in part (a) or
only states that gender does not matter and does not clearly state gender does not
matter and why the groups will still be sufficiently balanced.

If done well, the response can appeal to the randomization in part (a) (“divide
and proceed for each subgroup” vs. “divide and proceed”). A two factor design
that includes randomization rather than a block is considered partially correct.

Note, using the term “stratified random sample” is considered extraneous and is
not sufficient for an explanation of how randomization occurs in the new design.

Incorrect if all the response mentions is now having four groups of data to analyze. (No
randomization, not even a reference to the randomization in (a), and no clear
separation of the groups in the design.)

NOTE: If no randomization is specified anywhere, the highest possible score is a 2.

4 Complete Response

Both parts essentially correct.

3 Substantial Response
Part (a) is essentially correct and part (b) is partially correct.

OR

Part (a) is partially correct and part (b) is essentially correct.

Copyright © 2002 by College Entrance Examination Board. All rights reserved.


Advanced Placement Program and AP are registered trademarks of the College Entrance Examination Board.

10
火箭学院 火速提分
新页码 88

AP® STATISTICS
2002 SCORING GUIDELINES (Form B)

Question 3 (cont’d.)

2 Developing Response

Both parts are partially correct.

OR

Part (a) is essentially correct and part (b) is correct.

1 Minimal Response
Part (a) is partially correct and part (b) is incorrect.

OR

Part (a) is incorrect and part (b) is essentially or partially correct.

NOTE: PP is between a 1 and a 2. If only the before and after design is present,
it will be rounded down to a 1.

Copyright © 2002 by College Entrance Examination Board. All rights reserved.


Advanced Placement Program and AP are registered trademarks of the College Entrance Examination Board.

11
火箭学院 火速提分
新页码 89

AP® STATISTICS
2002 SCORING GUIDELINES (Form B)

Question 4

Solution

Part (a):

Large sample confidence interval for a population proportion.

Requires large population (N > 20n) and large sample. Can consider sample large since
npˆ = 1026(.39) = 400.14 ³ 10, n(1 - pˆ ) = 1026(.61) = 625.86 ³ 10

OR n p̂ > 5, n(1- p̂ ) > 5

pˆ (1 - pˆ )
OR pˆ ± 3 = .39 ± .0298 = (.3602,.4198) is within the interval (0, 1).
n

Computation of 95 percent confidence interval for one population proportion:

(.39)(.61)
.39 ± 1.96 = .39 ± .0298 = (.3602,.4198)
1026

If use the calculator with x = 400, get (.36002, .41971)

Interpretation of confidence interval:


Based on the sample, we estimate that the proportion of U.S. adults who would respond
“make some changes” is between .36 and .42.
OR
We are 95 percent confident that the proportion of all U.S. adults who would respond
“make some changes” is between .36 and .42.

Interpretation of the confidence level:


In the long run, 95 percent of the (different) intervals generated using this method will
contain the true population proportion.
OR
The method used to produce this estimate has a 5 percent error rate.

Copyright © 2002 by College Entrance Examination Board. All rights reserved.


Advanced Placement Program and AP are registered trademarks of the College Entrance Examination Board.

12
火箭学院 火速提分
新页码 90

AP® STATISTICS
2002 SCORING GUIDELINES (Form B)

Question 4 (cont’d.)

Part (b):

The statement is misleading because in addition to the 39 percent who wanted major change,
19 percent wanted a complete overhaul of the system. These two groups combined represent
58 percent (more than half) of the sample.

OR

Since the confidence interval from part (a) for the percentage of all U.S. adults who would
respond “make some major changes” includes 41 percent, we can’t conclude that the two
population percentages are different.

OR

The advocate is claiming that we should not change the system on the basis of adding the
30 percent who want to make some minor adjustments to the 11 percent who want to leave
the system the way it is now. It is misleading to claim that these 30 percent don’t want to
change the system.

Copyright © 2002 by College Entrance Examination Board. All rights reserved.


Advanced Placement Program and AP are registered trademarks of the College Entrance Examination Board.

13
火箭学院 火速提分
新页码 91

AP® STATISTICS
2002 SCORING GUIDELINES (Form B)

Question 4 (cont’d.)

Scoring

There are four parts to this question. You should read (2) and (3) together.

1. Determining the 95 percent confidence interval is


Essentially correct if it
• identifies the large sample confidence interval for a population proportion either by
name or by formula (numbers plugged in is OK);
• checks to make sure that the sample size is large enough; and
• and has correct computations (.3602, .4198)
Partially correct if just one of the above is incorrect or missing.

2. Interpretation of the confidence interval is


Essentially correct if it is clear, correct, and in context (include the numbers).
Partially correct if unclear or no context is given.
Incorrect if it says “The probability or chance that the true proportion is between .36 and
.42 is .95,” or if it looks like the sample proportion is in the confidence interval [“…the
proportion of U.S. adults who said…” (past tense)].
OK to say “95 percent sure.”

3. Interpretation of the confidence level is


Essentially correct if it is clear and correct (no context needed).
Partially correct if it is somewhat unclear but nothing is clearly incorrect.
Incorrect if it says “In 95 of every 100 samples, the population proportion will be
between .36 and .52,” or “will be in this interval.”

4. Answer to (b) may be scored essentially correct, partially correct, or incorrect.


For example, if the answer only says the 19 percent was ignored and does not make any
new comparisons or suggests a better calculation, then score this as a partial.

Essentially correct responses count as one part and partially correct responses count as ½ part. If
a paper is between two scores, use a holistic approach, considering overall strength and
communication, to determine whether the student gets the higher and lower score.

4 Complete Response
Four parts correct.

3 Substantial Response
Three parts correct.

2 Developing Response
Two parts correct.

1 Minimal Response
One part correct.

Copyright © 2002 by College Entrance Examination Board. All rights reserved.


Advanced Placement Program and AP are registered trademarks of the College Entrance Examination Board.

14
火箭学院 火速提分
新页码 92

AP® STATISTICS
2002 SCORING GUIDELINES (Form B)

Question 5

Solution

Part (a):

Part (b):

The range for faculty completion times is larger than for the students, but the IQR is the
same for both groups. The spread is similar in the middle 50 percent of the data, but the
smallest 25 percent and the largest 25 percent are more spread out for the faculty
members than for students.

Part (c):

Students should comment on at least two of center (mean, median, or general location),
variation, or shape. The statements should be correct and clear and suitable for the
school newspaper.

Example: Although some faculty negotiated the obstacle course quickly, in general
students tended to have shorter completion times. (location) The student completion
times, ranging from 3.75 minutes to 16.5 minutes, were more consistent than the faculty
times, which ranged from 4.5 minutes to 25 minutes. (variation) Many students and
faculty finished relatively quickly, but the slower half of each group tended to spread out.
(shape)

Copyright © 2002 by College Entrance Examination Board. All rights reserved.


Advanced Placement Program and AP are registered trademarks of the College Entrance Examination Board.

15
火箭学院 火速提分
新页码 93

AP® STATISTICS
2002 SCORING GUIDELINES (Form B)

Question 5 (cont’d.)

Scoring

Part (a) is considered

Essentially correct if the boxplots are drawn correctly with labels and scale given.

Partially correct if boxplots have no more than one error such as these: missing labels,
missing scales, not drawn to scale, or drawn showing outliers.

NOTES: • It is a considered a minus if "time" label is omitted.


• Any graphic that is not clearly a boxplot is incorrect.
Part (b) is considered

Essentially correct if response notes both that spread in the center is similar for the two
groups and that spread in the tails is greater for faculty than for students.

Partially correct if response only comments that variability is greater for the faculty than
for the students.

NOTE: It is considered a minus if the 25 is called an outlier.

Part (c) is considered

Essentially correct if there is a clear and coherent statement that comments on at least
two of center (mean, median, or general location), variation, or shape. Both faculty and
students must be mentioned in the response.

Partially correct if only comments on one aspect of the distribution or if communication


is weak.

Copyright © 2002 by College Entrance Examination Board. All rights reserved.


Advanced Placement Program and AP are registered trademarks of the College Entrance Examination Board.

16
火箭学院 火速提分
新页码 94

AP® STATISTICS
2002 SCORING GUIDELINES (Form B)

Question 5 (cont’d.)

4 Complete Response

Essentially correct on all three parts.

3 Substantial Response

Essentially correct on two parts and partially correct on the other part.

2 Developing Response

Essentially correct on two parts and incorrect on the other.

OR

Essentially correct on one part and partially correct on the other two parts.

OR

Partially correct on all three parts.

OR

Essentially correct on one part, partially correct on one part, and incorrect
on one part.

1 Minimal Response

Essentially correct on one part and incorrect on the other two parts.

OR

Partially correct on one or two parts and incorrect on the other.

Copyright © 2002 by College Entrance Examination Board. All rights reserved.


Advanced Placement Program and AP are registered trademarks of the College Entrance Examination Board.

17
火箭学院 火速提分
新页码 95

AP® STATISTICS
2002 SCORING GUIDELINES (Form B)

Question 6

Solution

Part (a):

Part 1: States a correct pair of hypotheses, with symbols defined.

µ1990 = mean number of lab classes for Biology majors in 1990


µ2000 = mean number of lab classes for Biology majors in 2000

H 0 : m1990 = m2000 H 0 : m1990 - m2000 = 0


OR
H 0 : m1990 ¹ m2000 H 0 : m1990 - m2000 ¹ 0

Part 2: Identifies a correct test (by name or by formula) and checks (not just lists)
appropriate assumptions.

x1990 - x2000 - 0 1.83 - 1.93


Two-sample t test t= =
s 21990 s 2 2000 1.292 1.37 2
+ +
n1 n2 200 200
Conditions:
1. Random samples – given in problem statement
2. Normal population distributions or large samples. Populations aren't
normal, but since n1 = 200 and n2 = 200 are both large, it is OK to
perform the t test.

OR
x1990 - x2000 - 0
Student may select the two-sample z test, using z =
s 21990 s 2 2000
+
n1 n2
as long as they comment on the large sample size.

Copyright © 2002 by College Entrance Examination Board. All rights reserved.


Advanced Placement Program and AP are registered trademarks of the College Entrance Examination Board.

18
火箭学院 火速提分
新页码 96

AP® STATISTICS
2002 SCORING GUIDELINES (Form B)

Question 6 (cont’d.)

Part 3: Correct mechanics, including the value of the test statistic, df, and P-value (or
rejection region).

N Mean StDev SE Mean


1990 200 1.83 1.29 0.091
2000 200 1.93 1.37 0.097

t = -.751 df = 396 P-value = .453

(or based on tables P-value > .20) Also OK to use conservative df of 199
OR Calculator using sample statistics:
t = .7515, P-value = .4527, df = 396.57
Calculator using raw data
t = .7512, P-value = .4529, df = 396.67
OR use a confidence interval approach:
Difference = mu 1990 - mu 2000
Estimate for difference: -0.100
90 percent CI for difference: (-0.319, 0.119)
OR use a “pooled” procedure
t = -.751 p=.452 df = 398
OR if use rejection region, t.05 = + 1.660

Part 4: States a correct conclusion in the context of the problem, linking to the result of
the statistical test.

Because the P-value > α, fail to reject H0. There is not convincing evidence that the
mean number of lab classes in 2000 is different than it was in 1990.

Part (b):

Answers question about (a): The test in (a) does not answer this question because the
distributions could be different even though the means are the same.

Part 1: States correct hypotheses.


H0: The distribution of number of lab classes is the same in 2000 as it was in 1990.
Ha: The distribution of number of lab classes is not the same in 2000 as in 1990.

Copyright © 2002 by College Entrance Examination Board. All rights reserved.


Advanced Placement Program and AP are registered trademarks of the College Entrance Examination Board.

19
火箭学院 火速提分
新页码 97

AP® STATISTICS
2002 SCORING GUIDELINES (Form B)

Question 6 (cont’d.)

Part 2: Identifies the correct test (by name or by formula) and checks (not just lists)
appropriate assumptions.
(observed - exp ected )2
Chi-square test (of homogeneity): χ 2 = å
exp ected

Conditions: Sample size is large. To check, must compute the expected counts:

1990 Expected 2000 Expected


24 24
67 67
59 59
19 19
21 21
10 10

All expected counts are greater than or equal to 5 (or 10), so it is OK to proceed with the
chi-square test.

Part 3: Correct mechanics.

Chi-Sq = 0.667 + 0.667 + 0.373 + 0.373 +


0.017 + 0.017 + 4.263 + 4.263 +
1.190 + 1.190 + 0.400 + 0.400 = 13.821
DF = 5, P-Value = 0.017

Part 4: States a correct conclusion in context using the result of the statistical test.

Because the P-value < α (problem states α = .10), reject H0. There is convincing
evidence that the distribution of number of lab classes is not the same in 2000 as it
was in 1990.

Copyright © 2002 by College Entrance Examination Board. All rights reserved.


Advanced Placement Program and AP are registered trademarks of the College Entrance Examination Board.

20
火箭学院 火速提分
新页码 98

AP® STATISTICS
2002 SCORING GUIDELINES (Form B)

Question 6 (cont’d.)

Part (c):

Although the two distributions are centered in about the same place, the shape of the
distributions is different. In 2000, the proportion of students taking three labs was
smaller than in 1990.

The statement must be backed up by appropriate graphs, such as two histograms (with
appropriate scales) or two boxplots.

Histograms

NOTE: If the student just uses the default intervals on the calculator, they will get odd looking
histograms. (This is a minus.)

Copyright © 2002 by College Entrance Examination Board. All rights reserved.


Advanced Placement Program and AP are registered trademarks of the College Entrance Examination Board.

21
火箭学院 火速提分
新页码 99

AP® STATISTICS
2002 SCORING GUIDELINES (Form B)

Question 6 (cont’d.)

Other acceptable graphs include:

Boxplots: Segmented Bar Graph

100%
90%
80%
70%
60%
50%
40%
30%
20%
10%
0%

1990 2000

Scoring

Part (a) is

Essentially correct if
All four parts of the test are essentially correct (except minor arithmetic errors).
Since there are so many parts, do not count off if degrees of freedom are omitted.
Not defining µ1 and µ2 is a minus.

Almost essentially correct if


Three parts are essentially correct and communication is very good.

Partially correct if
Two or three of the parts of the test are essentially correct.

NOTES:
1. Do not need to state that random samples are used since it is in problem statement.
2. If student checks on normality and comments that the histogram does not look
normal or that a boxplot has outliers and then says the t test is not appropriate (i.e.,
fails to recognize that it is OK to proceed with a t test based on the large sample size),
part two is not correct.
3. Mentioning samples are independent is a plus.

Part (b) is

Essentially correct if
The student recognizes that part (a) does not answer the question posed and has
all four parts of the chi-square test completely correct.

Copyright © 2002 by College Entrance Examination Board. All rights reserved.


Advanced Placement Program and AP are registered trademarks of the College Entrance Examination Board.

22
火箭学院 火速提分
新页码 100

AP® STATISTICS
2002 SCORING GUIDELINES (Form B)

Question 6 (cont’d.)

Almost essentially correct if


The student recognizes that part (a) does not answer the question posed and has three
parts essentially correct, including mechanics, and communication is very good.

Partially correct if
The student recognizes that part (a) does not answer the question posed and has
two or three parts of the chi-square test completely correct.

NOTES:
1. Part (b) is incorrect if the student says the test in (a) does not answer the question
posed. However, if the student rejects the null hypothesis in (a), a “yes” answer to
(b) could be correct.
2. If the student says, “No. We should do a chi-square test,” but doesn’t do one, this is a
plus.
3. If student uses the 1990 data as the expected counts and the 2000 data as the
observed counts and finds χ2=23.789, the student does not receive credit for parts two
or three (but can still get a P).

Part (c) is

Essentially correct if
response has a coherent description of the difference between the 1990 and 2000
distributions and gives appropriate graphs.

NOTE: If part (b) is incorrect because the student thinks that the test in (a) does
answer the question posed, but does good graphs and bases the argument in part
(c) on the graphs, part (c) can be scored as essentially correct.

Partially correct if
explanation is weak (such as describing the difference only in terms of
comparing the means) or missing but reasonable graphs are included
OR
incorrect graphs are included but the statements made about the distributions are
consistent with the given graphs.
OR
graphs are missing but analysis extends the results of (a) and (b).

NOTE: Part (c) should be considered incorrect if graphs that use the frequencies
as data are constructed.

If student falls between two scores, read the paper holistically, keeping in mind the
number of major errors and quality of communication (4 = complete response, 3 =
substantial response, 2 = developing response, 1 = minimal response).

Copyright © 2002 by College Entrance Examination Board. All rights reserved.


Advanced Placement Program and AP are registered trademarks of the College Entrance Examination Board.

23
火箭学院 火速提分
新页码 101

AP® STATISTICS
2002 SCORING GUIDELINES (Form B)

Question 6 (cont’d.)

4 Complete Response

Essentially correct on all 3 parts.

3 Substantial Response

Essentially correct on any two parts and partially correct or incorrect on the
other part.

2 Developing Response

Essentially correct on one part and partially correct on at least one of the
other two parts

OR

Partially correct on all three parts.

1 Minimal Response

Essentially correct on one part and incorrect on the other two

OR

Partially correct on one or two parts and incorrect on the other(s).

Copyright © 2002 by College Entrance Examination Board. All rights reserved.


Advanced Placement Program and AP are registered trademarks of the College Entrance Examination Board.

24
关注“火箭学院”公众号

获取第一手
“课程热点资讯”

扫描二维码添加“火箭队长”

即刻获得
“资料真题大礼包”

You might also like